Sie sind auf Seite 1von 48

Questions pratiques 1

1) Un patient effectue une séance d'exercices de réadaptation cardiaque de phase I (en hospitalisation). Le
kinésithérapeute doit
mettre fin à une activité de bas niveau si l'un des changements suivants se produit ?
1. La pression artérielle diastolique augmente jusqu'à 120 mm Hg.
2. La fréquence respiratoire augmente jusqu'à 20 respirations par minute.
3. La pression artérielle systolique augmente de 20 mm Hg.
4. La fréquence cardiaque augmente de 20 bpm.

2) Un patient est évalué pour un éventuel syndrome du canal carpien et un test de vitesse de conduction nerveuse est
effectué.
Parmi les résultats suivants, lesquels confirmeraient le PLUS fortement le diagnostic ?
1. Diminution de la latence au niveau du coude.
2. Diminution de la latence au niveau du canal carpien.
3. Augmentation de la latence au niveau du canal carpien.
4. Augmentation de la latence au niveau de l'avant-bras.

3) Lors de l'examen d'un patient ayant des antécédents d'abus d'alcool, le kinésithérapeute note que le patient présente
les caractéristiques suivantes
tremblements fins au repos et réflexes hyperactifs. Le patient signale des douleurs fréquentes dans le quadrant
supérieur droit. Parmi les signes supplémentaires suivants, lequel est le plus probable ?
1. Jaunisse
2. Hyperhidrose
3. Hypotension
4. Toux nocturne

4) Parmi les résultats d'examen suivants, lesquels sont attendus chez un patient qui a également subi un clonus de la
cheville ?
1. Un gros orteil en mouvement lorsqu'on caresse la plante du pied
2. Faiblesse des fléchisseurs plantaires de la cheville lors d'un test de force à une répétition
3. Absence de sensation à la palpation de la partie postérieure de la jambe à l'aide d'un instrument pointu/mou.
4. Hyporeflexie lorsque les réflexes tendineux profonds sont déclenchés dans la partie inférieure de la jambe

5) Lequel des sites de tests sensoriels suivants correspond à la racinenerveuse C7 ?


1. Face palmaire de l'auriculaire (5e doigt)
2. Face dorsale du majeur (3e doigt)
3. Face latérale du bras
4. Face médiale du bras

6) Un nourrisson de 3 mois a un mauvais contrôle de la ligne médiane de la tête. Lors de l'évaluation, le


kinésithérapeute note le visage
et observe que le nourrisson présente une limitation de la rotation cervicale vers la gauche et de la flexion latérale
cervicale vers la droite. Un rapport radiologique indique une fusion prématurée des sutures crâniennes du
nourrisson. Le nourrisson a très probablement 1. torticolis musculaire congénital droit.
2. torticolis musculaire congénital gauche.
3. hypomobilité de la facette cervicale droite.
4. hypomobilité de la facette cervicale gauche.

7) Un patient a subi une greffe de peau d'une épaisseur fractionnée pour une brûlure d'une épaisseur partielle au niveau
du membre supérieur. Le chirurgien a
a demandé des exercices d'amplitude de mouvement pour le patient. Actuellement, le patient est capable de bouger
activement le membre supérieur sur un tiers de l'amplitude de mouvement pour la flexion de l'épaule. Sur la base de
cette constatation, quelle est la mesure la plus appropriée à prendre par le kinésithérapeute à ce stade ?
1. Reportez tout exercice d'amplitude de mouvement jusqu'à ce que le patient soit en mesure de participer plus
activement.
2. Commencer les exercices d'assistance active à l'amplitude des mouvements.
3. Commencer l'entraînement à la mobilité au lit pour faciliter l'utilisation des membres supérieurs.
4. Poursuivre les exercices d'amplitude de mouvement.

8) Laquelle des modalités suivantes traite le mieux la cause de la tendinite calcifiante du tendon bicipital ?

PEAT
4
2 Questions pratiques

1. Courant interférentiel au niveau sensoriel à 80 Hz à 100 Hz


2. Iontophorèse avec de l'acide acétique à 60 mA/minute
3. Stimulation électrique pulsée à haut voltage à 200 pps
4. Diathermie avec une installation de traitement parallèle

9) Pour minimiser l'irritation de la peau lors de la stimulation électrique fonctionnelle des nerfs, un kinésithérapeute
doit utiliser :
1. intensité plus faible, distance interélectrode plus grande et électrodes plus grandes.
2. une intensité plus faible, une plus grande distance interélectrode et des électrodes plus petites.
3. une intensité plus élevée, une distance interélectrode plus faible et des électrodes plus petites.
4. une intensité plus faible, une distance interélectrode plus petite et des électrodes plus grandes.

10) Le test musculaire manuel des muscles du plancher pelvien d'une patiente révèle une note médiocre (2/5). Parmi les
positions suivantes, quelle est la MEILLEURE pour commencer à se renforcer ?
1. Décubitus dorsal
2. Debout
3. Assis
4. La marche

11) Lors d'un examen de la force du coude à l'aide d'un test musculaire manuel, un patient met l'avant-bras en supination
lors d'une tentative de flexion du coude. Parmi les muscles suivants, lequel est le plus susceptible d'effectuer la
majeure partie du travail ?
1. Biceps brachial
2. Brachial
3. Supinator
4. Brachioradialis

12) Lors de l'évaluation de l'équilibre d'un patient, un kinésithérapeute pousse doucement le patient vers l'arrière et
observe comment le patient se remet de la perturbation. Quelle stratégie le patient est-il le plus susceptible d'utiliser
pour corriger la perturbation ?
1. Genou
2. Hanche
3. Cheville
4. Marche

13) Laquelle des constatations suivantes décrit le mieux un remplissage capillaire normal ?
1. Vasodilatation de rebond après le glaçage
2. Pression artérielle de 120/76 mm Hg
3. Mesure de l'oxymétrie de pouls à 98
4. Blanchiment du lit de l'ongle avec retour de la couleur en <3 secondes

14) Laquelle des instructions suivantes est la PLUS appropriée pour apprendre à un patient atteint d'une quadriplégie C6 à
se transférer d'un fauteuil roulant à un tapis ?
1. Gardez les doigts tendus pour obtenir une base de soutien plus large.
2. Tourner la tête et les épaules dans la même direction que le mouvement de hanche souhaité.
3. Tourner la tête et les épaules dans la direction opposée au mouvement de hanche souhaité.
4. Gardez les deux mains près des genoux pour bloquer les coudes.

15) Un patient souffre d'une lésion de la moelle épinière qui a endommagé les segments sacrés et perturbé l'arc réflexe
sacré. Quelles sont les caractéristiques les plus probables du patient ?
1. Contrôle volontaire de la défécation
2. Contraction tonique du sphincter anal externe
3. Flaccidité de la musculature du plancher pelvien
4. Absence permanente du réflexe gastrocolique

16) Un patient qui se plaint d'une vision double présente un ptosis, un strabisme latéral et une pupille dilatée dans l'œil
gauche. Parmi les résultats des tests des nerfs crâniens suivants, lequel est le plus susceptible d'être anormal ?
1. Réflexe pupillaire à la lumière
2. Force des muscles faciaux

PEAT
4
Questions pratiques 3

3. Réflexe de la mâchoire
4. Sensation de douleur au niveau de la visage

17) Outre les précautions habituelles, quelle autre précaution le kinésithérapeute doit-il observer lorsqu'il travaille avec
un patient infecté par un staphylocoque doré résistant à la méthicilline ?
1. En vol
2. Stérile
3. Gouttelettes
4. Contact

18) Le kinésithérapeute positionne un patient pour un drainage postural. Pour drainer au mieux le segment postérieur
des deux lobes inférieurs, le patient doit être placé dans laquelle des positions suivantes ?
1. Couché, tête en bas à un angle de 45
2. En décubitus dorsal, sur une surface plane
3. En position latérale, la tête surélevée à un angle de 30°.
4. Assis, penché en avant

19) La bosse des doigts est le plus souvent associée à l'une des pathologies suivantes ?
1. Lymphoedème
2. Maladie pulmonaire
3. Insuffisance veineuse chronique
4. Syndrome douloureux régional complexe

20) Une patiente enceinte de 8 mois présente un diastasis recti abdominal avec une séparation de 4 cm. Lequel des
exercices suivants serait l'exercice initial le PLUS approprié pour renforcer les abdominaux en position couchée ?
1. Boucles de tronc
2. Levage de tête par crochet
3. Glissement des jambes par inclinaison du bassin
4. Abaissement bilatéral des jambes

21) Un patient atteint de la maladie de Parkinson vient d'être admis dans une unité de réadaptation. Le patient est
dépendant pour tous les transferts et a besoin de l'aide modérée d'une personne pour marcher 9,1 m avec un
déambulateur standard. Pour faciliter le transfert des activités, il convient d'informer la famille sur les transferts :
1. lors d'une visite à domicile après la sortie du patient.
2. juste avant la sortie du patient.
3. au début du programme de réadaptation.
4. lorsque la famille se sent prête à ramener le patient à la maison.

22) Un patient porteur d'une prothèse transfémorale droite pourra maintenir le genou en extension lors de la mise en
charge si le centre de gravité du corps est déplacé de manière à ce que la ligne gravitationnelle s'abaisse :
1. postérieurement à l'axe de l'articulation du genou droit.
2. latéralement à l'axe de l'articulation du genou droit.
3. en avant de l'axe de l'articulation du genou droit.
4. médial de l'axe de l'articulation du genou droit.

23) Lors d'un test musculaire manuel des fléchisseurs de la hanche en position assise, un patient présente une rotation
latérale (externe) avec abduction de la cuisse lorsqu'une résistance est appliquée. Le kinésithérapeute doit suspecter
une substitution musculaire de la part du patient :
1. sartorius.
2. le tenseur du fasciae latae.
3. le long adducteur.
4. semimembranosus.

24) Un kinésithérapeute lit que la fiabilité interjuges d'un nouveau dynamomètre manuel est de 0,93. Quelle est
l'interprétation la plus appropriée de cette valeur ?
1. Des scores similaires ont été obtenus pour un groupe de sujets lorsque différents thérapeutes ont mesuré les
sujets
2. Des scores similaires ont été obtenus pour un groupe de sujets lorsque le même thérapeute a répété les mesures
3. Des scores dissemblables ont été obtenus pour un groupe de sujets lorsque des thérapeutes différents ont

PEAT
4
4 Questions pratiques

mesuré les sujets.


4. Des scores dissemblables ont été obtenus pour un groupe de sujets lorsque le même thérapeute a répété les
mesures.

25) Un chercheur en kinésithérapie met au point une étude visant à comparer les différences d'amplitude des
mouvements dans deux groupes de patients ayant subi une arthroplastie totale du genou. Sur une période de 8
semaines, un groupe de patients reçoit une thérapie physique ambulatoire une fois par semaine et l'autre groupe
reçoit une thérapie physique ambulatoire trois fois par semaine. Dans cette étude, quelle est la variable dépendante ?
1. Amplitude des mouvements
2. Fréquence des visites
3. Arthroplastie totale du genou
4. 8 semaines

26) Parmi les stratégies d'enseignement suivantes, laquelle est la PLUS appropriée pour un patient âgé atteint de
démence légère ?
1. Utiliser l'auditif ou le visuelséparément.
2. Utiliser des métaphores pour renforcer les nouveaux concepts.
3. Établir un rythme d'enseignement cohérent.
4. Présentez un seul élément d'information nouveau à la fois.

27) Un patient souffrant d'une insuffisance veineuse chronique des extrémités inférieures est le plus susceptible de
présenter :
1. veines superficielles normales, absence d'œdème, d'ulcération et de plaques de gangrène autour des orteils.
2. dilatation des veines superficielles, œdème et ulcération de stase.
3. pas d'œdème, pouls dorsal faible, extrémités froides et glabres.
4. dilatation des veines superficielles et œdème aggravé par la position assise ou l'élévation des membres
inférieurs.

28) Un patient atteint de dystrophie musculaire a été débranché de la ventilation mécanique il y a un jour. Le patient est
actuellement incapable d'éliminer les sécrétions de manière autonome, bien qu'il ait reçu des instructions sur les
techniques d'hygiène broncho-pulmonaire (positionnement, percussion, secousses et vibrations). Laquelle des
interventions suivantes est la plus appropriée pour aider ce patient à éliminer les sécrétions ?
1. Aspiration nasotrachéale
2. Assistance costophrénique manuelle
3. Oxygène supplémentaire
4. Entraînement des muscles respiratoires

29) Un patient âgé a subi une arthroplastie totale de la hanche à la suite d'une fracture de la hanche consécutive à une
chute à son domicile. Le patient vivait seul et était auparavant capable d'effectuer toutes les activités de la vie
quotidienne de manière indépendante. Il a été admis dans un centre de soins infirmiers spécialisés pour une
rééducation. L'objectif du patient est de rentrer chez lui. Sa famille estime qu'il devrait être admis dans un
établissement de soins de longue durée pour sa propre sécurité, une fois la rééducation aiguë achevée. L'action la
plus appropriée du kinésithérapeute est la suivante :
1. informer la famille que le patient doit déterminer lui-même son environnement de sortie.
2. recommander une conférence d'équipe avec le patient et sa famille pour discuter des plans de sortie.
3. programmer une visite à domicile pour déterminer si des modifications du domicile sont nécessaires avant la
sortie de l'hôpital.
4. mettre en œuvre un plan de traitement dont l'objectif à long terme est le retour à domicile.

30) Parmi les raisons suivantes, laquelle est l'objectif PRIMAIRE de l'application des analyses statistiques à la
recherche sur un seul sujet ?
1. Justifier l'analyse visuelle des données présentées sous forme de graphiques
2. Déterminer si les changements observés sont réels ou s'ils sont le fruit du hasard
3. Préconiser des changements dans les méthodes d'intervention
4. Établir la crédibilité de la technique d'intervention

31) Lors du retrait d'un pansement humide à sec sur une plaie qui s'écoule, un kinésithérapeute observe que la peau
entourant immédiatement la plaie est macérée. Que devrait recommander le thérapeute pour les soins futurs de la
plaie ? 1. Continuer à utiliser le type de pansement actuel.

PEAT
4
Questions pratiques 5

2. Rendre le pansement plus absorbant.


3. Laissez le pansement en place plus longtemps entre les changements de pansement.
4. Passer à un pansement compressif.

32) Un patient de 50 ans a subi une réparation ouverte sans complication d'une rupture de la coiffe des rotateurs il y a
deux semaines. Le patient demande au kinésithérapeute quand son épaule redeviendra normale. Parmi les délais
d'obtention des résultats escomptés suivants, lequel répond le PLUS précisément à la question de ce patient ?
1. 3 semaines pour soulever un objet de 2,3 kg (5 lb)
2. 3 semaines pour dormir sur le côté impliqué
3. 3 mois pour soulever le membre supérieur au-dessus de la tête afin d'atteindre une armoire
4. 3 mois pour jouer au golf

33) Un kinésithérapeute se prépare à évaluer un patient qui a subi une réduction fermée avec fixation par plâtre pour une
fracture de la cheville il y a un jour et qui ne supporte actuellement aucun poids. Le patient signale une douleur au
pied de l'extrémité blessée. D'après le rapport du patient, laquelle des procédures suivantes le kinésithérapeute doit-
il effectuer ? 1. Examen de la démarche
2. Capacités de transfert
3. Remplissage capillaire des orteils
4. Examen de la tension artérielle

34) Parmi les mouvements suivants, quels sont ceux qui sont le plus limités par un schéma capsulaire de la hanche ?
1. Rotation médiane (interne) et abduction
2. Rotation latérale (externe) et abduction
3. Extension et adduction
4. Flexion et rotation latérale (externe)

35) Le parent d'un enfant de 4 ans atteint de myéloméningocèle souhaite obtenir des orthèses pour l'entraînement à la
marche de l'enfant. L'enfant présente une lésion au niveauneurologique L1 . Laquelle des orthèses suivantes est la plus
appropriée pour que le kinésithérapeute en discute avec les parents ?
1. Hanche-genou-cheville-pied avec hanches verrouillées
2. Reciprocating-gait
3. Genou-cheville-pied avec une bande pelvienne
4. Cheville-pied

36) Un patient atteint de polyarthrite rhumatoïde se présente à la kinésithérapie avec des signes d'atrophie musculaire,
des ecchymoses, des joues gonflées et un diagnostic d'ostéoporose. Parmi les médicaments suivants, lequel est le
plus susceptible d'être administré au patient ? 1. Pénicilline (Ampicilline)
2. Prednisone (Deltasone)
3. Acide acétylsalicylique (aspirine)
4. Sels d'or

37) Un exercice aérobique régulier d'une durée d'au moins 30 minutes devrait être le plus bénéfique pour réduire la
pathologie associée à l'une des conditions suivantes ?
1. Diabète de type 1
2. Diabète de type 2
3. Sclérose en plaques
4. Sclérose latérale amyotrophique

38) Un patient suit une thérapie physique pour une large plaie au bas de la jambe. Après 3 semaines de traitement, la
plaie ne présente plus de tissu nécrotique, mais un exsudat abondant est présent. Lequel des pansements suivants est
le plus approprié pour la plaie à ce stade ?
1. Gaze saturée de sérum physiologique
2. Film semi-perméable
3. Gaze imprégnée d'oxyde de zinc
4. Pâte hydrocolloïde

39) Un patient porteur d'une prothèse transtibiale droite décrit un gauchissement du genou droit dans la phase d'appui du
pied à plat (réponse à la mise en charge) de la marche. Parmi les affections suivantes, laquelle est la plus
susceptible d'être à l'origine de ce problème ?

PEAT
4
6 Questions pratiques

1. Flexion plantaire excessive


2. Coussin de talon rigide
3. Talon bas
4. Insertion excessive du pied

40) Laquelle des recommandations suivantes est la PLUS appropriée pour un patient souffrant de reflux gastro-
œsophagien ?
1. Restez assis pendant au moins 10 minutes après un repas.
2. S'allonger sur le côté droit avant d'effectuer les exercices.
3. Les exercices de soulèvement de la tête doivent être effectués en position verticale.
4. Les activités nécessitant une position couchée doivent être effectuées avant un repas.

41) Un kinésithérapeute termine l'examen d'un patient hospitalisé présentant de multiples comorbidités et ayant subi une
arthroplastie totale du genou il y a deux jours. Le thérapeute observe une décoloration bleuâtre des orteils sur
l'extrémité opérée. Cette constatation doit être documentée comme suit :
1. cyanose.
2. la thrombose veineuse profonde.
3. Maladie de Raynaud.
4. la pathologie cardiaque.

42) Un kinésithérapeute évalue une patiente de 70 ans qui signale l'apparition d'une douleur médio-thoracique après
avoir travaillé dans un jardin pendant plusieurs heures. Parmi les éléments d'anamnèse suivants, lequel doit
renforcer la suspicion de fracture par compression thoracique chez le patient ?
1. Dysfonctionnement de l'intestin et de la vessie
2. Tabagisme et utilisation prolongée de stéroïdes
3. Hypertension et diabète
4. Emphysème et traitement hormonal substitutif

43) Parmi les modalités suivantes, laquelle est la plus appropriée pour diminuer la douleur et augmenter l'extensibilité
des tissus avant la réalisation d'exercices actifs de la main chez un patient atteint de polyarthrite rhumatoïde ?
1. Pack de gel froid
2. Ultrasons par contact direct
3. Diathermie continue à ondes courtes
4. Bain de paraffine

44) Lequel des exercices de renforcement du tronc suivants est le MEILLEUR pour un patient afin d'aider à minimiser
les complications associées à l'ostéoporose ?
1. Flexion
2. Rotation
3. Extension
4. Flexion latérale

45) Un kinésithérapeute travaille sur la mobilité fonctionnelle progressive avec un patient qui a subi une colectomie
transverse il y a deux jours. Le patient a développé une faible fièvre. Quel est l'examen le plus approprié à réaliser
par le thérapeute avant de poursuivre l'intervention auprès du patient ?
1. Fréquence cardiaque
2. Pression artérielle
3. Fréquence respiratoire
4. Auscultation

46) Un patient entre dans un programme de réadaptation cardiaque. Le kinésithérapeute doit d'abord demander au
patient de.. :
1. décrire les aspects corrects de l'exercice démontré par le thérapeute.
2. énumérer les problèmes liés à de mauvaises habitudes alimentaires.
3. identifier les effets néfastes du tabagisme en ce qui concerne les maladies cardiaques.
4. décrire le type d'angine dont souffre le patient.

47) Un kinésithérapeute travaillant dans une agence de soins à domicile traite un patient atteint de diabète sucré. Le
patient signale qu'il ne prend plus d'insuline. La PREMIÈRE mesure à prendre par le thérapeute doit être de

PEAT
4
Questions pratiques 7

1. apprendre au patient la bonne technique d'injection de l'insuline.


2. contacter l'infirmière à domicile du patient.
3. dire à la famille du patient de communiquer cette information au médecin.
4. demander au patient d'effectuer un test de glycémie urinaire pendant que le thérapeute est à son domicile.

48) Un objectif mesurable pour un programme d'éducation communautaire sur les techniques d'exercice appropriées
serait que les participants.. :
1. comprendre l'importance d'une période d'échauffement suffisante.
2. citer cinq techniques d'étirement qui peuvent être utilisées lors de l'échauffement.
3. surmonter les problèmes des membres inférieurs et adhérer à un programme d'exercice régulier.
4. apprécier les effets de l'augmentation de l'intensité de l'exercice sur la fréquence cardiaque.

49) En montant les escaliers, un patient âgé se penche vers l'avant en augmentant la flexion de la hanche. Lequel des
muscles suivants est le plus utilisé dans cette posture en avant ?
1. Rectus fémoral
2. Tenseur du fasciae latae
3. Semitendinosus
4. Paraspin lombaire

50) Les auteurs d'un article de recherche décrivent un résultat d'étude favorable comme non significatif parce que les
résultats de l'étude n'étaient pas statistiquement significatifs. Le résultat non significatif serait le PLUS
probablement dû à de mauvaises méthodes de recherche si l'une des caractéristiques suivantes de l'étude était
présente ?
1. Taille réduite de l'échantillon
2. Faible variance au sein du groupe
3. 85% de puissance d'analyse
4. Niveau alpha contrôlé

51) Au fur et à mesure qu'un patient progresse dans l'apprentissage d'une nouvelle habileté motrice, il devrait y avoir
une augmentation du type de retour d'information parmi les suivants ?
1. Tactile
2. Visuel
3. Intrinsèque
4. Verbal

52) Un patient subit une épreuve d'effort sur tapis roulant. L'apparition de complexes QRS anormalement larges et
irrégulièrement espacés sur l'électrocardiogramme représente :
1. la dépolarisation ventriculaire.
2. les contractions ventriculaires prématurées.
3. la fibrillation auriculaire.
4. repolarisation auriculaire.

53) Alors qu'il travaille dans une clinique privée, un kinésithérapeute observe la chute d'un patient sur le parking à
l'extérieur du cabinet. Le patient souffre d'une grave lacération à l'avant-bras. Le thérapeute fixe un pansement
compressif sur la plaie, mais constate que le sang traverse le pansement et le bandage. Laquelle des actions
suivantes le thérapeute doit-il effectuer AU SUIVANT?
1. Surélevez le membre et appliquez une pression sur la plaie.
2. Retirez le pansement et le bandage et recommencez avec un bandage plus serré.
3. Appliquez des pansements et des bandages supplémentaires et exercez une pression sur l'artère brachiale.
4. Appelez le médecin du patient et organisez le transport pour les soins médicaux.

54) Un kinésithérapeute effectue un test de réflexe comme le montrent les photographies. La flèche indique la trajectoire
du stimulus appliqué. Les résultats du test sont les plus susceptibles d'indiquer :
1. a les lésions des nerfs périphériques.
2. a lésion des cellules de la corne antérieure.
3. a réponse normale.
4. une lésion de la moelle épinière.

55) La modification du travail (position debout) montrée sur la photographie est la plus appropriée pour un patient

PEAT
4
8 Questions pratiques

souffrant des pathologies suivantes ?


1. Sténose lombaire centrale
2. Thrombose veineuse profonde
3. Scoliose thoraco-lombaire
4. Bombement discal lombaire postéro-latéral

56) Un coureur de fond se présente à la kinésithérapie avec une douleur insidieuse au bas de la jambe. Les résultats de
l'examen du patient révèlent une faiblesse de la flexion des orteils et de l'inversion de la cheville. Le
kinésithérapeute soupçonne une atteinte vasculaire associée à cette présentation. Parmi les endroits suivants, lequel
est le plus susceptible de révéler une diminution des pouls artériels chez le patient ?
1. Face dorsale du pied
2. Postérieurement à la malléole latérale
3. Postérieurement à la malléole médiale
4. Face latérale du creux poplité

57) Au cours d'une séance d'intervention avec un patient souffrant d'hémiplégie récente, un kinésithérapeute prévoit de
se concentrer sur les transferts depuis et vers un fauteuil roulant, une baignoire et une voiture. Laquelle des
méthodes suivantes devrait être la plus efficace pour le maintien à long terme de ces compétences ?
1. Pratiquer les activités dans un ordre aléatoire.
2. Séquencez les activités du plus facile au plus difficile.
3. Laissez le patient déterminer l'ordre des activités.
4. Établir une séquence d'entraînement prévisible mais variable.

58) Un patient est allongé sur le dos, les hanches et les genoux en extension et les mains derrière la tête. Le patient est
capable de soulever la tête, les épaules et le thorax de la table de traitement mais n'est pas en mesure de s'asseoir
complètement. Quel muscle le kinésithérapeute doit-il cibler dans le cadre d'un programme de renforcement ?
1. Iliopsoas
2. Oblique abdominale externe
3. Quadratus lumborum
4. Grand droit de l'abdomen

59) Un patient présente un syndrome de Cushing induit par des médicaments. Lequel des problèmes physiologiques
suivants est une manifestation courante de ce trouble ?
1. Hypotension
2. Hypercalémie
3. Catabolisme musculaire
4. Diminution des protéines le métabolisme

60) Un patient de 90 ans souffrant d'une insuffisance cardiaque congestive chronique n'est pas ambulatoire et réside
dans une maison de retraite depuis un an. Le patient a été récemment admis à l'hôpital après un épisode de
déshydratation. Parmi les plans de soins respiratoires prophylactiques suivants, lequel est le plus approprié ?
1. Se tourner, tousser et respirer profondément toutes les 1 à 2 heures d'éveil.
2. Percussion et vibration vigoureuses 4 fois/jour
3. Vibrations douces avec le pied du lit surélevé 1 fois/jour
4. Drainage postural segmentaire à l'aide de positions standard tout au long de la journée

61) Laquelle des mobilisations articulaires suivantes serait la plus efficace pour améliorer la capacité d'un patient à
passer en position terminale ?
1. Glissement postérieur de l'astragale sur le tibia
2. Glissement postérieur du calcanéum sur l'astragale
3. Glissement plantaire de la 1ère phalange proximale sur les les métatarsien
4. Glissement dorsal de la 1ère phalange proximale sur le les métatarsien

62) Un kinésithérapeute examine la force musculaire d'un patient. Le patient est invité à déplacer sa jambe le long de la
trajectoire indiquée par la flèche sur la photographie et à maintenir la jambe en place pendant que le thérapeute
applique une résistance à la jambe. Le patient se déplace dans l'amplitude de mouvement indiquée et est capable de
supporter la résistance maximale. L'intervention devrait porter sur les problèmes suivants ?
1. Tensions des rotateurs médians (internes) de la hanche
2. Faiblesse du petit et du moyen fessier

PEAT
4
Questions pratiques 9

3. Correction du déplacement latéral (externe) du tronc vers la droite


4. Faiblesse du muscle piriforme

63) Un patient souffrant d'une lésion complète de la moelle épinière thoracique est assis dans un fauteuil roulant sur un
coussin fait sur mesure. Des activités de soulagement de la pression doivent être effectuées :
1. lorsque le patient présente des signes d'escarres.
2. toutes les 15 à 20 minutes.
3. toutes les 1 à 2 heures.
4. si le patient ne dispose pas d'un coussin approprié.

64) Parmi les observations suivantes concernant les membres inférieurs, laquelle est la plus susceptible d'être le
PREMIER signe d'une maladie vasculaire occlusive ?
1. Œdème des jambes et des chevilles
2. Décoloration brune au niveau des chevilles
3. Cyanose accrue lorsque les jambes sont dépendantes
4. Absence de poils sur les orteils

65) Un kinésithérapeute apprend à un patient récemment opéré du genou à utiliser une canne pour descendre des
escaliers sans rampe. Le patient est partiellement en charge à droite. Laquelle des méthodes pédagogiques suivantes
le thérapeute doit-il utiliser pour enseigner cette activité au patient ?
1. Le thérapeute se tient derrière le patient et ce dernier descend avec la canne et la jambe gauche en premier.
2. Le thérapeute se tient à côté du patient et celui-ci descend en utilisant d'abord la jambe droite, puis la canne.
3. Le thérapeute se tient en dessous du patient, et le patient descend avec la canne et la jambe droite en premier.
4. Le thérapeute se tient derrière le patient et celui-ci descend en commençant par la jambe gauche.

66) Un patient souffrant de lombalgie signale également une constipation, des nausées et vomissements occasionnels et
une perte de poids inexpliquée. La douleur s'aggrave lorsque le patient est en décubitus dorsal et diminue lorsqu'il
se penche en avant lorsqu'il est assis. Les sclérotiques sont jaunes. Les signes et les symptômes correspondent le
PLUS à l'un des troubles suivants ?
1. Appendicite
2. Cholécystite
3. Carcinome du pancréas
4. Syndrome du côlon irritable

67) Une rotation excessive vers le haut de l'omoplate droite est observée lorsque le patient tente d'effectuer une flexion
de l'épaule. Lequel des exercices suivants est le plus approprié pour aider à corriger la rotation excessive de
l'omoplate ?
1. Protraction de l'omoplate droite contre résistance avec le bras droit à 90° de flexion
2. Élévation bilatérale de l'omoplate avec les membres supérieurs à 180° de flexion
3. Pompes murales avec maintien isométrique en fin de course, coudes tendus.
4. Adduction bilatérale de l'omoplate avec les membres supérieurs en rotation médiale (interne) et en adduction
dans le dos.

68) Parmi les diagnostics suivants, lequel est le plus souvent associé à l'incontinence urinaire ?
1. Orchite
2. Cancer du testicule
3. Torsion testiculaire
4. Hyperplasie bénigne de la prostate

69) Un kinésithérapeute travaille sur les transferts d'un patient ayant subi un accident vasculaire cérébral du tronc
cérébral. Le patient présente une ataxie des quatre extrémités et un tonus extenseur élevé des membres inférieurs.
Le patient a un contrôle moyen à bon du tronc. Lequel des transferts suivants est le MEILLEUR pour ce patient ?
1. Squat pivot
2. Planche coulissante
3. Pivot debout
4. Tuque dépendante

70) Un kinésithérapeute souhaite examiner la relation entre les notes obtenues au test musculaire manuel des membres
inférieurs et cinq catégories classées de capacité d'ambulation fonctionnelle dans un groupe d'adultes plus âgés.

PEAT
4
10 Questions pratiques

Parmi les statistiques suivantes, laquelle est la plus appropriée pour tester cette relation ?
1. Test t par paires
2. Corrélation produit-moment de Pearson (r)
3. Mann-Whitney U test
4. Rho de Spearman (rs)

71) Dans une étude, un coefficient de corrélation de 0,30 a été trouvé pour la relation entre deux variables. Laquelle des
interprétations suivantes de ce résultat est la PLUS appropriée ?
1. Les variables sont faiblement corrélées.
2. Trente pour cent de la variabilité d'une variable peut être expliquée par l'autre variable.
3. Il n'y a pas de différences significatives entre les variables.
4. La valeur prédictive positive entre les variables est faible.

72) Laquelle des conditions suivantes est la cause la plus probable d'une réduction de la capacité vitale chez un patient
atteint d'une tétraplégie auniveau C5-C6?
1. Diminution de l'expansion antéro-latérale du thorax résultant d'une paralysie des muscles intercostaux externes
2. Incapacité du patient à générer une pression intrapleurale négative en raison d'un diaphragme dénervé
3. Position de repos relativement élevée du diaphragme résultant d'une paralysie des muscles abdominaux.
4. Diminution de l'élévation de la cage thoracique due à la paralysie des muscles scalènes antérieurs et sterno-
cléido-mastoïdiens.

73) Un kinésithérapeute place des électrodes sur un patient pour surveiller l'activité électromyographique de surface.
L'emplacement de l'électrode illustré sur la photographie est le plus approprié pour surveiller lequel des muscles
suivants ?
1. Tenseur du fasciae latae
2. Sartorius
3. Rectus fémoral
4. Petit fessier

74) Un kinésithérapeute soumet un patient à un programme de musculation des membres inférieurs. Le mode d'exercice
est une presse à deux jambes utilisant des poids libres. Après une semaine, le patient montre une augmentation de
4,5 kg du poids qu'il est capable de soulever. Quelle est la cause la plus probable de l'augmentation de la force du
patient ?
1. Hypertrophie des fibres musculaires
2. Adaptation neurologique
3. Hyperplasie du muscle fibres
4. Augmentation de la quantité d'actineet la myosine

75) Un kinésithérapeute envisage d'utiliser la phonophorèse dans le cadre d'un plan d'intervention. Laquelle des étapes
suivantes est la PREMIÈRE étape correcte dans le processus de prise de décision concernant l'utilisation de la
phonophorèse ?
1. Décrire les objectifs et les résultats thérapeutiques.
2. Sélectionnez l'agent de couplage et le médicament appropriés.
3. Décider de la dosimétrie en choisissant le mode et la fréquence appropriés.
4. Déterminer s'il existe des contre-indications.

76) Un kinésithérapeute évalue un patient qui souffre de douleurs à l'épaule. Le patient remarque la douleur à l'épaule au
travail lorsqu'il stocke des étagères situées au-dessus de sa tête. La douleur n'est pas apparente lorsqu'il s'agit de
ranger des étagères au niveau de la taille ou de la poitrine. Le patient souffre très probablement d'une faiblesse dans
lequel des muscles suivants ?
1. Pectoralis mineur
2. Trapèze supérieur
3. Deltoïde
4. Rhomboïde majeur

77) Un patient ayant subi un léger accident vasculaire cérébral il y a trois semaines est préparé par un kinésithérapeute
en vue d'une sortie à domicile et d'un programme de jour pour adultes. Pour faciliter le plan de sortie, le
professionnel de la santé le plus approprié à consulter est le thérapeute :
1. coordonnateur des soins infirmiers qualifiés.

PEAT
4
Questions pratiques 11

2. ergothérapeute.
3. l'assistant social médical.
4. médecin principal.

78) Dans une étude de recherche, un kinésithérapeute effectue la même mesure goniométrique sur le même groupe de
sujets de contrôle au cours de deux séances de test consécutives. Ce processus est utilisé pour démontrer laquelle
des propriétés de mesure suivantes ?
1. Validité externe
2. Fiabilité de l'instrument
3. Fiabilité intraréférentielle
4. Fiabilité inter-juges

79) La stimulation électrique au niveau sensoriel est la plus appropriée pour un patient souffrant de l'une des
pathologies suivantes ?
1. Lombalgie chronique d'origine somatique
2. Entorse aiguë de la cheville avec œdème
3. Tendinite du sous-épineux
4. Syndrome de Raynaud actif

80) Parmi les diagnostics suivants, quel est celui qui bénéficierait le PLUS d'une respiration par les lèvres pincées
pendant l'exercice ?
1. Maladie vasculaire périphérique
2. Insuffisance cardiaque congestive
3. Emphysème
4. Sarcoïdose

81) Un patient a des antécédents de douleurs cervicales qui sont aggravées par de longues périodes en position assise et
qui s'aggravent progressivement le soir. L'amplitude des mouvements et la force du cou et des épaules se situent
dans les limites normales. La sensibilité et les réflexes sont intacts dans les deux extrémités supérieures. Le patient
a la tête en avant et une cyphose thoracique excessive. Le programme d'exercices le plus approprié doit être axé sur
:
1. étirement des fléchisseurs du cou et renforcement des pectoraux.
2. renforcement des trapèzes supérieurs et étirement des pectoraux.
3. renforcement des pectoraux et étirement des rhomboïdes.
4. renforcement du rhomboïde et rétraction axiale du cou.

82) Un patient suspecté de souffrir d'hypoglycémie est le plus susceptible de présenter les signes suivants ?
1. Une haleine fruitée
2. Soif, nausées et vomissements
3. Muqueuses sèches et croûteuses
4. Difficultés d'élocution et de concentration

83) Un patient est limité dans l'abduction de l'épaule, comme le montre la radiographie présentée. Laquelle des
techniques de mobilisation suivantes est la plus susceptible d'aider le patient à augmenter l'abduction de l'épaule ?
1. Glissement postérieur
2. Glissement antérieur
3. Glissement inférieur
4. Glisse supérieure

84) Parmi les tâches suivantes, laquelle est la plus appropriée pour un kinésithérapeute de déléguer à un bénévole ?
1. Réapprovisionner les cabines de traitement en linge, gel à ultrasons et lotion de massage.
2. S'occuper d'un patient qui est sur une table basculante pendant que le thérapeute prend un appel téléphonique.
3. Transporter un patient qui se plaint de vertiges jusqu'à sa chambre
4. Transfert d'un patient de la table à matelas à un fauteuil roulant

85) Un kinésithérapeute enseigne une habileté motrice à un patient souffrant d'hémiplégie chronique. Parmi les
approches pédagogiques suivantes, laquelle devrait être la plus privilégiée ?
1. L'accoutumance
2. Sensibilisation

PEAT
4
12 Questions pratiques

3. Compensation stratégie
4. Récupération des mouvements normaux

86) Un patient souffre de douleurs dans le bas du dos et dans les jambes, les symptômes s'étendant jusqu'à la plante du
pied. Au cours de l'examen de kinésithérapie, le patient ne signale pas de douleur à la jambe dans la première
position de test (photographie n° 1), mais signale une augmentation importante des symptômes dans la deuxième
position de test (photographie n° 2). Laquelle des conclusions suivantes est la plus probable ?
1. La douleur est due à une hernie discale.
2. Le patient peut présenter des symptômes non organiques.
3. Les symptômes sont le résultat d'une inflammation du nerf sciatique.
4. Les ischio-jambiers sont dans un état de contraction facilitée.

87) Un patient est adressé pour obtenir des recommandations concernant l'achat d'un fauteuil roulant. Les mesures du
patient en position assise sont de 40,6 cm au point le plus large des hanches et de 45,7 cm de l'arrière des fesses au
pli poplité. Parmi les dimensions de fauteuil roulant suivantes, quelle est celle qui conviendrait le mieux aux
besoins de ce patient ?
1. Siège largeur et profondeur d'assise de 45,7 cm (18 in)
2. Siège largeur de 18 pouces (45,7 cm) et la profondeur de l'assise de 16 en (40.6 cm)
3. Siège largeur et profondeur d'assise de 40,6 cm (16 in)
4. Siège largeur de 16 pouces (40,6 cm) et la profondeur de l'assise de 18 en (45.7 cm)

88) Un kinésithérapeute commence à intervenir auprès d'un patient postopératoire qui prend 20 mg d'oxycodone par
voie orale (Oxycontin) pour soulager sa douleur. En planifiant une intervention pour ce patient, le kinésithérapeute
doit anticiper que : 1. le patient peut être hypertendu.
2. le médicament peut déclencher des arythmies cardiaques chez le patient.
3. le patient est plus susceptible de développer une diarrhée.
4. le patient peut présenter une dépression respiratoire.

89) Dans lequel des scénarios suivants les gants doivent-ils être utilisés pour respecter les précautions standard ?
1. Pendant tous les soins prodigués aux patients en milieu hospitalier
2. Réalisation d'une amplitude de mouvement sur un patient atteint du syndrome d'immunodéficience acquise
3. Masser le cou d'un patient atteint d'hépatite C
4. Changer la couche d'un nourrisson en milieu pédiatrique

90) Laquelle des structures suivantes est indiquée par la flèche sur la radiographie ?
1. Sillon intertuberculaire
2. Grand tubercule
3. Petit tubercule
4. Apophyse coracoïde

91) Laquelle des descriptions suivantes correspond à une réaction NORMALE du patient au test du réflexe tendineux
profond ? 1. 2 ou plus (+)
2. 3 ou plus (++)
3. 0 ou moins (-)
4. 1 ou moins (-)

92) Un patient présente une faiblesse de tout le membre inférieur droit avec une force normale du côté gauche. Les tests
sensoriels montrent une perte de la sensation de pression sur la cuisse et la jambe droites et une perte de la sensation
de douleur et de température sur la cuisse et la jambe gauches. Le patient présente un signe de Babinski positif à
droite. Parmi les résultats associés suivants, lequel est le plus susceptible d'être trouvé lors d'un examen plus
approfondi de ce patient ?
1. La présence d'un clonus dans la cheville gauche
2. Atrophie marquée des muscles du membre inférieur droit
3. Spasticité du membre inférieur gauche
4. Augmentation des réflexes tendineux profonds du côté droit

93) Alors qu'un kinésithérapeute effectue un entraînement au transfert du lit à la chaise avec un patient qui a subi une
arthroplastie totale du genou il y a deux jours, le moniteur électrocardiographique s'alarme et le thérapeute note que
des contractions ventriculaires prématurées se sont développées. Quelle est la MEILLEURE ligne de conduite

PEAT
4
Questions pratiques 13

pour le thérapeute à ce stade ?


1. Placez le patient sur une surface stable et arrêtez la thérapie physique pour la journée.
2. Poursuivez le transfert vers le fauteuil et surveillez les niveaux de saturation en oxygène du patient.
3. Poursuivez le transfert vers le fauteuil et informez immédiatement l'infirmière de la présence d'un ventricule
prématuré.
les contractions.
4. Placez le patient sur une surface stable et déterminez la stabilité des contractions ventriculaires prématurées.

94) Laquelle des positions de drainage postural suivantes est la PLUS appropriée pour un patient qui souffre d'une
pneumonie par aspiration dans le lobe moyen droit et qui a subi une craniotomie il y a deux jours ?
1. Position latérale droite avec un quart de tour vers le décubitus dorsal
2. Position latérale gauche avec un quart de tour vers le décubitus dorsal, la tête du lit étant abaissée d'environ
20°.
3. Position latérale droite avec un quart de tour vers le décubitus dorsal, la tête du lit étant abaissée d'environ 20°.
4. Position latérale gauche avec un quart de tour vers le décubitus dorsal

95) Pour aider les étudiants à appliquer une compétence nouvellement acquise à la pratique clinique, l'action la plus
efficace pour l'instructeur clinique est la suivante :
1. indiquer des situations cliniques possibles et discuter de la manière dont la compétence s'y appliquerait.
2. demander aux élèves de rechercher des documents de référence et de dresser une liste des étapes nécessaires à
l'acquisition de la compétence.
3. préparer une liste d'indications et de contre-indications pour la compétence.
4. demander aux étudiants de fournir des exemples de situations cliniques dans lesquelles la compétence serait
appliquée de manière appropriée.

96) Lors de l'examen de l'épaule d'un patient, un kinésithérapeute note la présence d'un motif capsulaire sans douleur
radiculaire. Pour aider à déterminer la cause du schéma capsulaire, le thérapeute doit procéder à un examen NEXT:
1. effectuer une compression axiale de la colonne cervicale pour vérifier la présence d'une compression de la
racine nerveuse.
2. demander au patient s'il a déjà subi un traumatisme de l'articulation de l'épaule.
3. vérifier la présence d'un arc douloureux pendant l'amplitude active des mouvements.
4. examiner l'épaule à la recherche d'une rupture de la coiffe des rotateurs.

97) Un patient a été opéré du genou il y a 4 semaines. Parmi les paramètres de stimulation électrique neuromusculaire
suivants, lequel serait le PLUS approprié pour renforcer le muscle quadriceps du patient ?
1. 1 à 4 pps, 100 microsecondes
2. 1 à 4 pps, 350 microsecondes
3. 40 à 50 pps, 350 microsecondes
4. 100 pps, 100 microsecondes

98) On demande à un patient de saisir des deux mains une carte blanche entre le pouce ( 1er doigt) et l'index (2e doigt). Le
kinésithérapeute tire sur la carte dans le sens de la flèche indiquée. Lorsque le thérapeute tire sur la carte, le pouce
droit du patient fléchit au niveau de l'articulation interphalangienne. Les résultats indiquent une faiblesse dans
lequel des muscles suivants ?
1. Flexor pollicis longus
2. Abducteur pollicis brevis
3. Adducteur pollicis
4. Extensor pollicis longus

99) Lors de l'évaluation de la marche d'un patient, un kinésithérapeute remarque que le patient se penche latéralement de
manière excessive vers le côté droit pendant la phase de demi-poussée sur la droite. Pour que le thérapeute puisse
tester la force normale du muscle suspecté (5/5), le patient doit D'ABORD être positionné :
1. en position couchée, le genou tendu.
2. en décubitus dorsal avec le genou plié.
3. assis avec la hanche fléchie à 110°.
4. en se couchant sur le côté gauche.

100) Un patient ayant subi un accident cérébelleux a bénéficié d'un entraînement fonctionnel à l'équilibre pendant 4
semaines. Lequel des tests suivants est le plus approprié pour mesurer l'efficacité de l'intervention de kinésithérapie

PEAT
4
14 Questions pratiques

?
1. Test de Romberg
2. Balance Berg
3. Évaluation Fugl-Meyer
4. Indice de Barthel

101) Une patiente au huitième mois de grossesse présente un engourdissement et des picotements de la main gauche, à
l'exception de l'auriculaire (5e doigt). Elle présente un œdème de la main et des doigts, un signe de Tinel positif au
niveau du poignet et une bonne note (4/5) au test musculaire des fléchisseurs du poignet et des doigts.
L'intervention la plus appropriée est la suivante :
1. une attelle de poignet pour positionner le poignet en extension complète.
2. une compresse chaude suivie d'exercices de glissement du tendon.
3. exercices de résistance pour les fléchisseurs du poignet et des doigts.
4. repos et élévation fréquents du membre supérieur gauche.

102) Lequel des emplacements suivants correspond à la fonction sensorielle de la racine nerveuse sortant entre la
troisième et la quatrième vertèbre lombaire ?
1. Latéral ligne d'articulation du genou
2. Médiale ligne d'articulation du genou
3. Plantaire aspect du talon
4. Espace web dorsal entre le premier et le deuxième orteil

103) Un service de kinésithérapie pour patients hospitalisés ne dispose que d'un kinésithérapeute et d'un assistant
kinésithérapeute en raison d'une maladie du personnel. Le traitement d'un patient présentant les conditions et
circonstances suivantes est le plus approprié pour que le thérapeute le délègue à l'assistant.
1. Ataxie, qui fait l'objet d'un essai visant à déterminer un dispositif d'assistance approprié
2. Hémiparésie, dont l'évaluation initiale n'est pas terminée
3. Sclérose en plaques, qui reçoit un entraînement à la marche à l'aide d'un déambulateur à roulettes
4. Maladie d'Alzheimer, qui est facilement agitée et qui reçoit un entraînement initial à la marche

104) Lorsque l'on envisage des tests spéciaux pour l'évaluation orthopédique, il est important de choisir un test dont la
validité est suffisante, car les tests de l'orthopédie ne sont pas nécessairement les mêmes que ceux de l'orthopédie :
1. les résultats du test peuvent être standardisés.
2. Le test mesure ce qu'il est censé mesurer.
3. les résultats du test sont reproductibles.
4. le test peut être effectué avec précision par quelqu'un d'autre.

105) Un patient qui a récemment subi une amputation transtibiale bilatérale souhaite faire construire une rampe pour
pouvoir se déplacer en fauteuil roulant de la terrasse arrière de sa maison jusqu'à la piscine. La distance verticale
entre la porte et le sol est de 1,5 m. Laquelle des spécifications de rampe suivantes est la MEILLEURE pour le
patient ?
1. Une rampe continue, 60 ft (18 m) de long
2. Une rampe continue, 30 ft (9 m) de long
3. Deux rampes de 60 pieds chacune (18 m) de long, reliés par a niveau zone
4. Deux rampes de 30 pieds chacune(9 m) de long, reliés par a niveau zone

106) Un kinésithérapeute élabore un programme d'exercices pour une personne âgée afin d'améliorer le maintien de sa
santé et de sa condition physique. Le patient a des antécédents de fracture de la cheville droite il y a 15 ans, de
réparation d'un anévrisme aortique il y a 3 ans et de chirurgie à cœur ouvert pour un remplacement valvulaire il y a
6 ans. Laquelle des activités suivantes est CONTRE-INDIQUÉE?
1. Exercice aquatique pendant 30 minutes
2. Cyclisme à 60 % de la fréquence cardiaque maximale
3. Exercice d'endurance à l'aide d'une gymnastique de base
4. Lever des poids à 50 % de la contraction volontaire maximale

107) Lequel des programmes à domicile suivants est le PLUS approprié pour un patient souffrant d'épicondylite latérale
chronique ?
1. Utilisation d'une manchette d'avant-bras pour augmenter la charge sur les tendons extenseurs
2. Exercices de renforcement et d'étirement du poignet

PEAT
4
Questions pratiques 15

3. Administration d'une iontophorèse avec de la dexaméthasone (Decadron) et de la lidocaïne (Xylocaïne)


4. Massage par friction du tendon brachioradialis

108) Un patient reçoit une traction cervicale intermittente mécanique avec un cycle de travail marche/arrêt de 20
secondes/10 secondes. Le patient fait état d'une douleur accrue à chaque fois que l'unité de traction se met en
marche, qui s'atténue ensuite pendant la durée de la mise en marche. Parmi les modifications suivantes du cycle de
travail, laquelle est la plus appropriée ?
1. Augmenter le temps d'arrêt à 20 secondes.
2. Augmenter le temps de marche à 25 secondes.
3. Diminuer le temps d'arrêt à 5 secondes.
4. Diminuer le temps de marche à 15 secondes.

109) Laquelle des mesures suivantes des gaz du sang artériel indique une acidémie ?
1. No table of contents entries found.

110) Un patient atteint d'un cancer du testicule est le plus susceptible de signaler les signes ou symptômes suivants AU
DÉBUT?
1. Mal de dos
2. Douleur référée à l'aine
3. Gonflement de la lymphe superficielle nœuds
4. Douleur et gonflement testiculaires diffus

111) Laquelle des positions suivantes est la MEILLEURE pour évaluer la longueur du muscle droit fémoral d'un
patient ?
1. Position latérale avec la hanche testée en flexion
2. En décubitus dorsal avec la hanche et le genou testés en flexion
3. En position couchée, le genou testé étant en flexion
4. Position latérale avec la hanche testée en extension

112) Quelle est la position fermée du poignet ?


1. Flexion avec déviation ulnaire
2. Flexion avec déviation radiale
3. Extension avec déviation ulnaire
4. Extension avec déviation radiale

113) Un patient est tombé en faisant de l'escalade il y a deux jours. La chute a entraîné une fracture de la cheville droite,
qui a nécessité une réduction ouverte et une fixation interne. Le patient a également subi une lésion nerveuse qui a
entraîné une faiblesse importante des muscles dumyotome droitC6-C7. Le patient est limité à la mise en décharge du
membre inférieur droit. Parmi les aides techniques suivantes, laquelle est la plus appropriée pour le patient ?
1. Béquilles axillaires
2. Fauteuil roulant
3. Déambulateur standard
4. Béquilles d'avant-bras

114) Un kinésithérapeute examine les pieds d'un patient atteint de diabète de type 2. Lequel des tests suivants est BEST
pour déterminer le risque de développement d'une ulcération du pied chez le patient ?
1. Sensation de douleur
2. Seuil de pression
3. Discrimination en deux points
4. Sensibilisation à la température

115) Un kinésithérapeute planifie une série de trois sessions de formation sur l'évaluation kinésithérapeutique du bas du
dos pour les étudiants en médecine de quatrième année. Parmi les activités suivantes, laquelle est la plus importante
à inclure dans la première session ?
1. Une vue d'ensemble du matériel qui sera couvert dans les trois sessions
2. Un pré-test pour déterminer le niveau de connaissance des participants
3. Participation active des étudiants à une évaluation du bas du dos
4. Démonstration complète d'une évaluation du bas du dos

PEAT
4
16 Questions pratiques

116) Un kinésithérapeute lit dans un rapport qu'un enfant a un score standard de -2,0 sur une mesure du développement.
Parmi les interprétations suivantes de ce score, laquelle est la MEILLEURE?
1. Les le score de l'enfant est bien inférieur au score moyen, comparé aux scores des autres enfants.
2. Les le score de l'enfant est proche du score moyen, comparé aux scores des autres enfants.
3. Les a de meilleurs résultats qu'environ 85 % des enfants.
4. Les a de moins bons résultats qu'environ 85 % des enfants.

117) Un patient souffrant d'une herniepostéro-latérale du nucleus pulposus L4-L5est le plus susceptible de présenter des déficits
sensoriels dans laquelle des localisations suivantes ?
1. Genou médial
2. Au-dessus du creux poplité
3. Face plantaire du cinquième orteil
4. Dorsale du gros orteil

118) Un patient de 72 ans signale une douleur bilatérale au mollet qui s'aggrave progressivement pendant la marche. Les
épisodes douloureux ont progressivement augmenté en fréquence et en intensité, après un début insidieux il y a
deux ans. La position assise diminue les symptômes du patient. Parmi les pathologies suivantes, quelle est la cause
la plus probable de la douleur à la jambe du patient ?
1. Sténose foraminale lombaire
2. Claudication de l'artère fessière
3. Hernie du noyau pulpeux
4. Thrombose veineuse profonde

119) Un patient atteint du syndrome de Guillain-Barré et souffrant d'une paralysie progressive est admis dans une unité
de soins intensifs. Parmi les interventions suivantes, laquelle est la plus appropriée pour un kinésithérapeute en tant
que membre de l'équipe interdisciplinaire qui gère les soins du patient ?
1. Modifier les réglages de la ventilation mécanique du patient.
2. Élaborer le calendrier des médicaments du patient.
3. Effectuer les techniques de dégagement des voies respiratoires.
4. Discuter avec la famille du pronostic médical du patient.

120) Un kinésithérapeute examinant le jeu de l'articulation du poignet constate une restriction dans la direction indiquée
par la flèche. Le thérapeute doit suspecter une diminution du mouvement de quelle articulation ?
1. Écart radial
2. Déviation du cubitus
3. Flexion
4. Extension

121) Parmi les complications du système nerveux suivantes, laquelle indiquerait le pronostic le plus défavorable pour un
patient atteint du syndrome d'immunodéficience acquise (SIDA) ?
1. Toxoplasmose
2. Leucoencéphalopathie
3. Myélopathie
4. Polyneuropathie

122) Parmi les résultats d'examen suivants, lequel correspond à une déshydratation ?
1. Mauvaise turgescence
2. Œdème dépendant
3. Œdème de Quincke
4. Hypertension

123) Une patiente, serveuse, signale une faiblesse de la main droite qui l'amène à laisser tomber des plats au travail
chaque fois qu'elle est distraite. Le patient signale également des douleurs au poignet et à la main ainsi qu'un
engourdissement au niveau de l'éminence thénar, qui surviennent tous deux pendant la journée et la nuit. La force
musculaire thénar du patient est faible (2/5) sur l'ensemble du corps. Parmi les objectifs thérapeutiques à long terme
suivants, lequel est le PLUS approprié pour ce patient ?
1. L'objectif est d'augmenter la sensibilité tactile en 4 semaines, afin que le patient puisse tenir des objets et ne
pas faire tomber la vaisselle au travail.
2. Le patient doit démontrer une diminution de la douleur dans le poignet afin de pouvoir dormir la nuit.

PEAT
4
Questions pratiques 17

3. L'objectif à long terme est d'augmenter la force des muscles du thénar et de réduire la douleur du poignet à des
niveaux normaux.
4. Le patient démontrera une force musculaire thénar de bonne qualité (4/5) dans 6 semaines et sera capable de
tenir des objets dans la main au travail.

124) Un patient atteint d'une lésion cérébelleuse grave peut s'asseoir de manière autonome, se tenir debout avec une aide
minimale et a besoin d'une aide modérée de la part d'une personne pour marcher en toute sécurité. Le patient
souhaite passer de manière autonome d'un fauteuil roulant à un lit. Laquelle des techniques de transfert suivantes est
la plus appropriée pour le patient ?
1. Pivot debout
2. Pivot manuel
3. Ascenseur mécanique
4. Glissant

125) Un kinésithérapeute examine un patient qui souffre d'une douleur unilatérale des membres inférieurs lors de la
marche. La douleur est soulagée par le repos. Laquelle des descriptions suivantes du comportement des symptômes
rapportés par le patient aidera le thérapeute à confirmer le diagnostic de claudication intermittente ?
1. Soulagement de la douleur en cas de flexion vers l'avant ou d'assise
2. Soulagement de la douleur en position debout
3. Douleur par crampes survenant à une distance prévisible de la marche
4. Engourdissements et fourmillements qui se produisent à une distance prévisible.

126) Un patient se plaint de vertiges et d'une vision floue lorsqu'il marche et tourne la tête vers la gauche ou la droite. Le
patient n'a aucun problème lorsque la tête est maintenue immobile pendant la marche. Parmi les systèmes suivants,
lequel est le plus susceptible d'être impliqué ?
1. Visuel
2. Vestibulaire
3. Somatosensoriel
4. Appareil locomoteur

127) Un kinésithérapeute prépare l'entraînement à la marche d'un jeune adulte paraplégique. Laquelle des options
d'entraînement à la marche suivantes est la plus appropriée pour la première séance du patient ?
1. Modèle de marche avec déambulateur
2. Modèle de marche avec béquilles de l'avant-bras
3. Modèle de marche avec béquilles axillaires
4. Modèle d'oscillation et de marche dans les barres parallèles

128) Un kinésithérapeute traite un patient qui souffre d'une limitation chronique de l'amplitude des mouvements en raison
d'un resserrement des ischio-jambiers. Parmi les applications suivantes des ultrasons et des étirements, laquelle est
la MEILLEURE pour rétablir l'amplitude normale des mouvements ?
1. Ultrasons pulsés à 1 MHz avec apparition de l'étirement 10 minutes après le traitement par ultrasons
2. Ultrasons pulsés à 1 MHz avec étirements pendant 10 minutes pendant et immédiatement après les ultrasons.
traitement
3. Ultrasons continus à 1 MHz avec apparition de l'étirement 10 minutes après le traitement par ultrasons.
4. Ultrasons continus à 1 MHz avec étirements pendant 10 minutes pendant et immédiatement après les ultrasons.
traitement

129) Au cours d'une première évaluation en kinésithérapie, un patient signale un essoufflement occasionnel. Lequel des
rapports suivants du patient indique la nécessité PRIMAIRE d'un examen pulmonaire plus approfondi par le
kinésithérapeute ?
1. Il y a une dyspnée nocturne paroxystique.
2. Les symptômes sont soulagés en se penchant en avant.
3. Il y a eu un changement récent dans les activités physiques.
4. L'essoufflement est associé à des étourdissements.

130) Un patient de 90 ans, hospitalisé, a été orienté vers une évaluation et une intervention de kinésithérapie à la suite
d'unefracture de la C5 consécutive à une chute. Le patient décrit des douleurs au cou et au genou gauche, mais ne signale
aucun autre changement post-lésionnel au niveau des extrémités. Le pronostic de réadaptation du patient doit
PRIMORITAIREMENT être basé sur : 1. niveau de fonction antérieur.

PEAT
4
18 Questions pratiques

2. l'amplitude des mouvements du genou gauche.


3. l'utilisation d'un collier cervical.
4. sensorielle du membre supérieurl'intégrité.

131) Lors de l'examen d'un patient, le kinésithérapeute observe visuellement les signes de la maladie de Charcot. Parmi
les signes ou symptômes suivants, lesquels sont les plus susceptibles d'être présents et de correspondre à ce
diagnostic ?
1. Erythème
2. Douleur aiguë ou lancinante dans l'avant-pied
3. Sensibilité accrue au toucher
4. Ulcère ouvert sur la tête du deuxième métatarsien

132) Un kinésithérapeute examine le dossier médical d'un patient en unité de soins intensifs. Le patient a été admis la nuit
précédente au service des urgences après un accident de moto ayant entraîné une fracture du fémur droit. Le
thérapeute note que le médecin a prescrit un examen Doppler de la jambe gauche. Le thérapeute doit
1. poursuivre l'évaluation et l'intervention sans aucun changement.
2. suspendre la kinésithérapie jusqu'à ce que les résultats de l'étude soient obtenus et interprétés par le médecin.
3. poursuivre l'évaluation et limiter l'intervention au transfert sur une chaise de chevet.
4. obtenir l'autorisation de l'infirmière pour intervenir auprès du patient.

133) Un kinésithérapeute travaille avec un patient ambulatoire qui a eu un accident vasculaire cérébral et qui vit
actuellement dans un centre d'aide à la vie autonome. Laquelle des affirmations suivantes est l'objectif fonctionnel
le PLUS approprié pour ce patient ?
1. Le patient pourra enfiler une orthèse cheville-pied avec de l'aide.
2. Le patient marchera de manière autonome 165 pieds (50 m) avec une canne droite de sa chambre à la cafétéria.
3. Le patient aura une force normale (5/5) des muscles quadriceps.
4. L'équilibre du patient s'améliorera jusqu'à ce qu'il puisse se tenir debout sur le membre inférieur concerné
pendant 20 secondes.

134) Un kinésithérapeute examine un patient souffrant d'une cardiopathie congestive et prenant des diurétiques. Parmi les
affections suivantes des membres inférieurs, laquelle est la plus probable ?
1. Couleur pâle
2. Peau brillante
3. Œdème de Quincke
4. Absence de croissance des cheveux

135) Un kinésithérapeute examine le système tégumentaire d'un patient et note une zone d'escarre noire et durcie à
l'extrémité du premier orteil. Le thérapeute conclura très probablement que le patient souffre de l'une des affections
suivantes ?
1. Engelures
2. Diabète
3. Seule une lésion superficielle de la peau à cet endroit est constatée
4. Au moins sur toute l'épaisseur des lésions de la peau à cet endroit

136) Un kinésithérapeute prescrit un programme d'exercices pour aider un patient à perdre du poids. Le patient est atteint
de diabète de type 1. Il faut conseiller au patient de ne commencer une séance d'exercice physique que si sa
glycémie est 1. inférieur à 60 mg/dL.
2. supérieure à 250 mg/dL.
3. entre 60 et 100 mg/dL.
4. entre 100 et 250 mg/dL.

137) Lors de la planification d'un programme de kinésithérapie pour les patients gériatriques, il convient de tenir compte
d'un changement important lié à l'âge :
1. l'incapacité d'apprendre de nouvelles tâches motrices.
2. une diminution de la sensation de douleur.
3. une baisse de motivation.
4. l'incapacité de sélectionner d'autresles stratégies de mouvement.

138) Un kinésithérapeute examine le système tégumentaire d'un patient présentant une pigmentation cutanée foncée et

PEAT
4
Questions pratiques 19

remarque un approfondissement de la couleur de la peau sur la face postérieure gauche du calcanéum. Laquelle des
constatations suivantes indiquerait une escarre de stade I à cet endroit ?
1. Diminution de la température de la peau par rapport aux tissus environnants
2. Augmentation de la température de la peau par rapport aux tissus environnants
3. Blanchiment sous pression avec retour à la coloration précédente dans les 30 secondes suivant le retrait de la
pression.
4. Blanchiment avec pression appliquée qui revient à la coloration précédente 1 heure après le retrait de la
pression

139) Un kinésithérapeute étudie les effets de trois traitements de l'ataxie. Les patients ont été recrutés dans une clinique
locale et ont été assignés au hasard à l'une des trois conditions de traitement. Des mesures fonctionnelles ont été
prises 6 mois après le début du traitement. Lequel des tests statistiques suivants est le plus approprié pour l'analyse
des données de cette étude ?
1. Analyse factorielle de la variance
2. Analyse de variance à sens unique
3. Test t de l'échantillon indépendant
4. Test t à mesures répétées

140) Un kinésithérapeute évalue les nerfs crâniens d'un enfant atteint d'un médulloblastome. L'œil droit de l'enfant dévie
vers le milieu. L'enfant présente une atteinte de quel nerf crânien parmi les suivants ?
1. Oculomoteurs (III)
2. Trochléaire (IV)
3. Abducens (VI)
4. Vagus (X)

141) Un kinésithérapeute se prépare à apprendre à un patient atteint du syndrome de Guillain-Barré à se transférer d'un
fauteuil roulant à une table d'examen. Le patient pèse 68 kg et a une force médiocre (2/5) dans les deux extrémités
inférieures et une force moyenne (4/5) dans l'extrémité supérieure gauche. Le membre supérieur droit a une force
normale (5/5). Lequel des transferts assistés suivants est le plus approprié pour la première séance du patient ?
1. Ascenseur pour deux personnes sur le côté droit
2. Transfert de la planche coulissante vers le côté droit
3. Levage hydraulique du fauteuil roulant au tapis
4. Transfert complet debout-pivot vers le côté droit

142) Un kinésithérapeute travaille avec un patient souffrant d'une lésion de la moelle épinière. Pour prouver que le
patient a été éduqué sur les soins de la peau, le thérapeute doit noter que le patient.. :
1. ne peut tolérer plus d'une heure en fauteuil roulant.
2. a une bonne force des abaisseurs de l'omoplate et des extenseurs du coude.
3. est capable de citer trois causes de dégradation de la peau.
4. est capable de faire 10 pompes en fauteuil roulant.

143) L'augmentation du volume résiduel est le moins susceptible d'être constatée lors de l'examen de la fonction
pulmonaire d'un patient souffrant de l'une des pathologies suivantes ?
1. Atelectasie
2. Bronchectasie
3. Bronchite chronique
4. Emphysème

144) Pendant la grossesse, lequel des exercices suivants est CONTRE-INDIQUÉ?


1. Flexion des jambes
2. Ponts
3. Levées de jambes doubles
4. Respiration profonde avec expiration forcée

145) Un patient à qui l'on apprend une nouvelle tâche motrice est le plus susceptible d'avoir besoin d'une attention totale
à la tâche et d'un feedback extrinsèque pendant quelle phase de l'apprentissage moteur ?
1. Cognitif
2. Associatif
3. Autonome

PEAT
4
20 Questions pratiques

4. Cadre

146) Un patient récemment blessé à la cheville signale une douleur antérieure à la cheville lors de la marche. La douleur
est reproduite UNIQUEMENT lors de la rotation passive de la jambe inférieure. Parmi les lésions suivantes, quelle
est celle que le patient a le plus probablement subie ?
1. Entorse de la syndesmose
2. Entorse calcanéocuboïdienne
3. Fracture du cinquième métatarsien
4. Fracture de stress du tibia

147) Pour éviter l'apparence d'un mouvement accru, quel mouvement DOIT être évité lors de la mesure goniométrique
de l'abduction de l'épaule ?
1. Rotation vers le haut de l'omoplate
2. Rotation médiale (interne) de l'épaule
3. Rotation latérale (externe) de l'épaule
4. Flexion latérale du tronc vers le côté opposé

148) Lequel des tests de la fonction pulmonaire suivants mesure la quantité totale moyenne d'air déplacée pendant
l'inspiration chez un patient respirant normalement ?
1. Capacité vitale
2. Capacité inspiratoire
3. Volume courant
4. Volume de réserve inspiratoire

149) Un kinésithérapeute évalue un patient porteur d'un drain thoracique. Le thérapeute renverse accidentellement le
réservoir de collecte. Le thérapeute doit remettre le réservoir en position verticale et.. :
1. l'accrocher à une perche intraveineuse.
2. le placer sur la table de chevet du patient.
3. le suspendre sur le côté du lit.
4. contacter le médecin.

150) Un kinésithérapeute examine un patient souffrant d'une radiculite lombaire supérieure. Parmi les combinaisons de
mouvements suivantes, laquelle est la plus susceptible de reproduire la douleur associée à la radiculite ?
1. Extension du genou en décubitus dorsal avec flexion de la hanche
2. Flexion du genou en décubitus dorsal avec flexion de la hanche
3. Flexion du genou en décubitus dorsal avec extension de la hanche
4. Extension du genou en décubitus dorsal avec extension de la hanche

151) Un patient souffrant d'une déficience qui limite la flexion de la hanche à 105° bénéficiera le PLUS des appareils
d'assistance et des modifications suivantes pour accomplir les activités de la vie quotidienne ?
1. Hauteur d'assise surélevée
2. Pince à long manche
3. Diminution de la hauteur des escaliers
4. Aide aux chaussettes

152) Un patient atteint d'hémiparésie présente un pied tombant pendant la phase d'oscillation de la marche. L'application
la PLUS appropriée de la stimulation électrique fonctionnelle serait de traiter à la fois le tibialis anterior et lequel
des muscles suivants ?
1. Gastrocnémien
2. Tibialis postérieur
3. Extensor digitorumlongus
4. Fibulaire (péroné) longus

153) Un kinésithérapeute envisage d'étudier l'effet des compresses froides sur l'amplitude des mouvements passifs dans
deux groupes de sujets : un groupe expérimental et un groupe de contrôle. Pour le groupe expérimental, la
compresse froide sera appliquée sur les muscles ischio-jambiers une fois par jour pendant 5 jours. Laquelle des
méthodes expérimentales suivantes est la PLUS appropriée pour cette étude, afin de comparer l'effet des
compresses froides entre les deux groupes ?
1. Pour le groupe expérimental, recueillez les données relatives à l'amplitude des mouvements avant chaque

PEAT
4
Questions pratiques 21

traitement.
2. Pour les deux groupes, mesurer l'amplitude des mouvements au jour 5.
3. Pour les deux groupes, mesurer l'amplitude des mouvements aux jours 1 et 5.
4. Pour le groupe expérimental, mesurer l'amplitude des mouvements tous les jours. Pour le groupe témoin,
mesurer l'amplitude des mouvements aux jours 1 et 5.

154) Un kinésithérapeute travaille avec un patient qui sait qu'il est en phase terminale. Quelle est l'intervention la plus
appropriée lorsque le patient souhaite parler du pronostic ?
1. Décourager les discussions sur la mort ou le décès.
2. Orienter le patient vers un conseil pastoral.
3. Relater les expériences du thérapeute avec d'autres patients.
4. Encourager le patient à exprimer ses sentiments.

155) Un kinésithérapeute examine un patient qui ne s'est pas rendu compte de l'existence d'un ulcère circulaire sur la
surface plantaire de la deuxième tête métatarsienne. Le patient présente des orteils en marteau. Quels sont les
résultats supplémentaires les plus probables ?
1. Pouls artériel normal
2. Transpiration accrue au niveau du pied
3. Douleur accrue lors de l'élévation du pied
4. Modification de la biomécanique lors de la mise en charge

156) Un kinésithérapeute examinant le jeu de l'articulation du poignet constate une restriction dans la direction indiquée
par la flèche. Pour remédier à cette restriction, le thérapeute doit inclure une intervention visant à augmenter le
mouvement de l'index (2e doigt). 1. Flexion
2. Extension
3. Enlèvement
4. Rotation

157) Un kinésithérapeute traite un patient atteint de diabète sucré. Parmi les informations suivantes, laquelle est la plus
importante à inclure dans l'information du patient sur les bénéfices de l'exercice physique ?
1. La glycémie augmente pendant l'exercice.
2. Les exercices de musculation sont préférables aux exercices d'aérobic pour les patients diabétiques.
3. L'exercice physique peut réduire la quantité d'insuline que le patient doit s'administrer.
4. Il est recommandé aux patients diabétiques de faire de l'exercice avant les repas.

158) Un patient présente une douleur, un œdème et une sensibilité au niveau du bord médial de la main. Le patient
présente également des changements de couleur et de température de la peau, une hyperhidrose et une raideur
articulaire progressive du poignet et de la main. La cause la plus probable des signes et symptômes du patient est :
1. la discopathie cervicale.
2. Phénomène de Raynaud.
3. le syndrome douloureux régional complexe.
4. le syndrome du canal carpien.

159) Un kinésithérapeute planifie un programme d'éducation des patients pour un groupe de patients souffrant de
lombalgie chronique. Parmi les stratégies suivantes, laquelle doit être utilisée par le thérapeute pour augmenter le
PLUS possible la probabilité que les patients utilisent les mécanismes corporels appropriés qu'ils ont appris ?
1. Demandez aux patients de montrer qu'ils utilisent les bons mécanismes corporels.
2. Fournir une liste de référence d'articles décrivant la mécanique du corps.
3. Demandez aux patients de décrire les actions qu'ils font et qui augmentent leur mal de dos.
4. Fournir des informations sur la fréquence des lésions lombaires dues à une mauvaise mécanique corporelle.

160) Un kinésithérapeute doit s'attendre à ce qu'un patient décrive la douleur associée à un antécédent de polyarthrite
rhumatoïde depuis 5 ans comme suit :
1. douleur matinale avec raideur qui s'améliore avec l'activité.
2. douleur qui s'aggrave au début d'une activité.
3. une douleur constante qui dure toute la journée.
4. une douleur qui augmente tout au long de la journée.

161) L'utilisation d'un massage avec de la glace sur la (les) zone(s) affectée(s) est la plus appropriée pour un patient qui

PEAT
4
22 Questions pratiques

présente les problèmes suivants ?


1. Point de déclenchement dans le muscle trapèze supérieur
2. Spasticité des muscles fléchisseurs de la plante après un accident vasculaire cérébral
3. Endolorissement des muscles abdominaux à l'effort
4. Diminution de l'extensibilité des muscles ischio-jambiers bilatéraux

162) Un patient se présente avec une faiblesse soudaine des muscles faciaux du côté droit. Le patient est incapable de
plisser le front, de sourire, de froncer les lèvres ou de plisser le nez. Il n'y a pas de larmoiement dans l'œil droit du
patient, il y a une diminution de la sensation de goût sur le côté droit de la langue et une sécheresse de la bouche. Le
réflexe cornéen du patient est absent à droite mais normal à gauche, et les sensations de piqûre et de température
sont normales des deux côtés du visage. Cette présentation est caractéristique de : 1. névralgie du trijumeau.
2. Paralysie de Bell.
3. cortical gauche accident vasculaire cérébral.
4. oculomoteur des lésions nerveuses.

163) Laquelle des interventions suivantes est la PLUS appropriée pour un patient atteint de polyarthrite rhumatoïde
juvénile qui souffre d'un gonflement douloureux des deux genoux ?
1. Exercices de résistance
2. Les étirements pour prévenircontractures
3. Exercices doux et actifs
4. Programme de marche

164) Un kinésithérapeute se demande si les résultats d'une étude peuvent être utilement appliqués à d'autres contextes de
pratique que celui dans lequel l'étude a été menée. Lequel des types de validité suivants est le plus important dans
ce scénario ?
1. Interne
2. Construire
3. Concurrent
4. Externe

165) Le traitement d'un patient hémophile présentant une hémarthrose subaiguë du genou doit INITIALEMENT
comprendre :
1. active assistanteamplitude de mouvementl'exercice à la genou.
2. l'instruction du patient pour la mise en charge jusqu'à la tolérance.
3. résistif douxamplitude de mouvementl'exercice à lagenou.
4. immobilisation continue du genou dans une attelle d'extension.

166) Un patient souffrant d'une lésion cérébrale traumatique est renvoyé chez lui à l'issue d'une rééducation en milieu
hospitalier. Lequel des outils d'évaluation suivants doit être utilisé pour évaluer AU MIEUX le potentiel du
patient ?
1. Échelle de résultats de Glasgow
2. Évaluation Fugl-Meyer
3. Échelle des niveaux de fonctionnement cognitif de Rancho Los Amigos
4. Profil d'impact de la maladie

167) Parmi les résultats d'examen suivants, lequel est le plus susceptible d'être positif chez un patient souffrant d'un
syndrome de la cauda équina ?
1. Clonus de la cheville
2. Signe de Babinski
3. Rétention urinaire
4. Test de flexion du genou en position couchée positif

168) Un patient souffrant de lombalgie suit un traitement depuis deux séances. Au cours de la séance de traitement
actuelle, le patient dit au kinésithérapeute que la douleur se centralise avec les exercices d'extension, mais qu'elle est
aussi intense que lors de la première séance de traitement. Le patient est frustré par cette réaction. La réponse la
plus appropriée du thérapeute est la suivante :
1. poursuivre le programme actuel.
2. éliminer les exercices d'extension.
3. consulter le médecin du patient au sujet de la situation.

PEAT
4
Questions pratiques 23

4. passer aux exercices de flexion du tronc.

169) Un patient bénéficie d'une intervention de kinésithérapie pour une polyarthrite rhumatoïde en rémission. Laquelle
des interventions suivantes est la plus appropriée pour le patient ?
1. Étirements contracter-relaxer pour les structures tendues
2. Techniques de mobilisation des extrémités
3. Élimination des activités fonctionnelles des parties concernées
4. Exercices de renforcement pour les muscles faibles

170) Un patient souffrant d'une douleur thoracique due à une ischémie myocardique présentera très probablement :
1. douleur accrue à la palpation de la paroi thoracique.
2. douleur accrue lors d'une respiration profonde.
3. soulagement avec l'ingestion de nitroglycérine (Nitrostat).
4. soulagement avec l'ingestion d'antiacides.

171) Lors d'un examen de la posture, le kinésithérapeute constate que les deux rotules du patient sont orientées vers
l'intérieur lorsqu'on les regarde de face. La cause la plus probable de ce problème est un excès : 1. antéversion
fémorale.
2. faiblesse du vaste médial.
3. genu varum.
4. torsion tibiale médiane.

172) Laquelle des affirmations suivantes ne documente PAS le résultat pour le patient ?
1. Au bout de 4 semaines, le patient se déplaçait seul dans son fauteuil roulant.
2. Le patient a démontré qu'il était capable d'exécuter un programme à domicile de manière indépendante après 2
semaines.
3. Le patient a marché 30,5 m avec une aide minimale au bout d'une semaine.
4. Le patient a suivi des séances de kinésithérapie 3 fois par semaine pendant 2 semaines.

173) Un patient a été blessé par balle à la colonne vertébrale dans la région de L1. Le patient présente une faiblesse du
membre inférieur gauche et une incapacité à bouger le genou, la cheville ou le pied. Les réflexes du tendon rotulien
et du tendon d'Achille du patient sont augmentés du côté gauche. Le patient présente également une perte de
proprioception au niveau de la cheville et du genou gauches, un signe de Babinski positif du côté gauche et une
diminution de la sensibilité à la piqûre d'épingle et aux changements de température au niveau de la cuisse, de la
jambe et du pied droits. Les résultats de tous les examens des nerfs crâniens du patient sont normaux. Ces résultats
sont cohérents avec les lésions suivantes ?
1. Sectionnement complet de la moelle épinière
2. Lésion de la corne antérieure gauche de la moelle épinière
3. Lésion du côté gauche de la moelle épinière
4. Lésion de la zone centrale de la moelle épinière

174) Parmi les interventions suivantes, quelle est celle qui répondrait le PLUS spécifiquement aux complications
attendues de la radiothérapie dans la région pelvienne ?
1. Etirement de la hanche antérieure
2. Se mettre en écharpe en toussant
3. Renforcement progressif des abdominaux
4. Exercices de stabilisation du bassin

175) Parmi les types de patients suivants, lequel devrait bénéficier du programme d'exercices à domicile le plus détaillé et
le plus long ?
1. Un patient présentant des symptômes aigus
2. Un patient présentant des symptômes d'irritabilité
3. Un patient qui a une bonne conscience de son corps
4. Un patient qui suit des séances de kinésithérapie une fois par semaine

176) Un kinésithérapeute mesure le volume expiratoire forcé en 1 seconde (VEMS) d'un patient souffrant d'une maladie
pulmonaire obstructive légère et détermine un rapport VEMS/CVF(capacité vitale forcée) de 60 %. Le thérapeute doit
reconnaître que le patient
1. a unVEMS normal.

PEAT
4
24 Questions pratiques

2. devra faire l'objet d'une surveillance attentive pendant l'exercice aérobique.


3. ne doivent pas faire d'exercice au-delà de 60 % de la fréquence cardiaque maximale prévue du patient.
4. ne doit pasfaire de l'exercice aérobique sans l'accord du médecin.

177) Parmi les changements de couleur de la peau suivants, lequel est le plus susceptible d'être le signe d'une maladie du
foie ?
1. Couleur pâle
2. Jaunisse
3. Cyanose
4. Hyperpigmentation

178) Le syndrome d'immunodéficience acquise est le plus susceptible d'être présent chez un patient présentant l'une des
affections hématologiques suivantes ?
1. Compte de CD4 de 150 cellules/mL
2. Titre élevé d'anticorps antinucléoprotéines
3. Titre élevé d'antigène leucocytaire humain
4. Nombre de neutrophiles inférieur à 5000 cellules/mm3

179) Un patient atteint de leucémie a développé une thrombocytopénie après une greffe de moelle osseuse. Laquelle des
mesures suivantes est indicative de l'état de la thrombocytopénie ?
1. Nombre delymphocytes T4
2. Numération des globules rouges
3. Numération plaquettaire
4. Numération des globules blancs

180) Lequel des facteurs suivants est considéré comme un facteur de risque primaire d'athérosclérose ?
1. Le stress
2. Obésité
3. Le tabagisme
4. Mode de vie sédentaire

181) Laquelle des affirmations suivantes est un objectif à court terme correctement rédigé ?
1. En deux semaines, le patient augmentera sa distance de marche de 15 m à 30 m avec un déambulateur à roues.
2. Le patient marchera 30 m avec un déambulateur à roulettes et une aide minimale dans 3 semaines.
3. Dans 4 semaines, le patient marchera avec une aide minimale sur une distance de 30 m sans perte d'équilibre.
4. Le patient augmentera sa distance de marche de 50 à 100 pieds (15 à 30 m) avec un déambulateur à roues de
manière autonome d'ici la sortie de l'hôpital.

182) Lequel des nerfs suivants innerve les muscles du plancher pelvien ?
1. Sciatique
2. Pudendal
3. Fessier inférieur
4. Génitofémorale

183) Un patient souffrant d'une polyarthrite rhumatoïde d'apparition récente est le plus susceptible de signaler les
symptômes suivants ?
1. Intolérance à la chaleur
2. Malaise et fatigue
3. Céphalées de tension
4. Nausées et diarrhées

184) Pour un patient atteint de diabète de type 1 qui suit un programme d'entraînement cardiovasculaire, quel
changement dans la prise en charge du diabète est le plus susceptible d'être mis en place au fur et à mesure de
l'amélioration de la condition physique ?
1. Passage à un médicament oral plutôt qu'injectable
2. Diminuer l'apport calorique pendant les 2 à 3 heures qui suivent les séances d'exercice.
3. Diminution de la quantité d'insuline prise quotidiennement
4. Augmentation de la quantité d'insuline prise quotidiennement

PEAT
4
Questions pratiques 25

185) Un kinésithérapeute évalue un patient qui a subi un accident vasculaire cérébral et qui présente un mauvais contrôle
du pied et de la cheville. Lorsqu'on lui demande de lever davantage le pied pendant la phase intermédiaire de la
marche, le patient dorsiflexe la cheville avec une éversion excessive. Pour corriger ce problème, le biofeedback
électromyographique facilitateur est le plus susceptible d'être utilisé avec lequel des muscles suivants ?
1. Extensor digitorum longus
2. Tibialis anterior
3. Fibulaire (péroné) court
4. Flexor hallucis longus

186) Quel changement cutané associé au vieillissement a le plus grand effet sur la cicatrisation des plaies ?
1. Réduction des sensations
2. Diminution de l'élasticité de lapeau
3. Diminution de la prolifération épidermique
4. Changement de pigmentation

187) Parmi les questions suivantes, laquelle est la PLUS appropriée pour un outil de mesure des résultats fonctionnels ?
1. Jusqu'à quelle hauteur pouvez-vous lever le bras ?
2. Depuis combien de temps avez-vous cette douleur ?
3. Combien de fois marchez-vous en boitant ?
4. A quel point cette jambe vous gêne-t-elle ?

188) Un patient souffre d'une capsulite adhésive de l'articulation de l'épaule. L'examen de l'amplitude des mouvements
révèle une restriction de la rotation latérale (externe) et de l'abduction de l'épaule. Laquelle des procédures de
mobilisation suivantes doit être effectuée pour le patient PREMIÈREMENT?
1. Glissement postérieur
2. Distraction
3. Glissement antérieur
4. Rotation latérale (externe)

189) Un patient souffrant du syndrome du piriforme droit est adressé à la kinésithérapie pour évaluation et intervention.
Le patient a subi une arthroplastie totale de la hanche droite il y a deux ans. Laquelle des interventions suivantes
nécessite des précautions supplémentaires pour ce patient ?
1. Stimulation électrique transcutanée des nerfs
2. En continu échographie
3. Paquets de chaleur
4. Massage à la bonne hanche

190) Un patient adulte impliqué dans un accident de la route a subi de multiples traumatismes, notamment des côtes
fracturées du côté droit. Le patient est inconscient, intubé et sous ventilation mécanique dans l'unité de soins
intensifs. La radiographie pulmonaire montre l'apparition d'un infiltrat dans le lobe inférieur droit au cours des deux
derniers jours. Des crépitants et une respiration sifflante sont entendus dans les champs pulmonaires inférieurs
droits. Parmi les programmes de kinésithérapie thoracique suivants, lequel est le plus approprié ?
1. Hyperventilation et aspiration manuelles en position latérale gauche
2. Positionnement en décubitus dorsal pour l'aspiration, suivi d'une hyperventilation manuelle en position latérale
gauche
3. Aspiration, percussion et vibration en position latérale droite
4. Positionnement sur le côté gauche uniquement pour les exercices de respiration profonde

191) Le style d'apprentissage d'un étudiant en kinésithérapie est décrit comme une préférence pour l'expérimentation
active et l'expérience concrète. Pour maîtriser l'évaluation des dysfonctions musculo-squelettiques, laquelle des
stratégies éducatives suivantes serait la MOINS efficace pour le kinésithérapeute qui supervise l'étudiant ?
1. Pratiquer et fournir un retour d'information sur les compétences en matière d'examen
2. Jeu de rôle sur le processus d'entretien
3. Assigner des lectures sur le dysfonctionnement
4. Réalisation de l'évaluation en mode autoguidage

192) Un kinésithérapeute planifie un traitement d'hydrothérapie en immersion totale. Ce traitement est CONTRE-
INDIQUÉ pour laquelle des conditions suivantes ?
1. Ostéoporose

PEAT
4
26 Questions pratiques

2. Œdème périphérique
3. induite par l'exerciceasthme
4. Hypertension non contrôlée

193) Lors de l'examen d'un patient, un kinésithérapeute constate une bradycardie. Le patient signale une constipation et
une intolérance au froid. Parmi les pathologies suivantes, quelle est la cause la plus probable de ce signe et de ces
symptômes ?
1. Hypothyroïdie
2. Hyperthyroïdie
3. Hyperparathyroïdie
4. Hypoparathyroidsim

194) Un patient ayant subi une lésiontraumatique de la moelle épinièreau niveau C6 il y a un mois signale une céphalée
violente et une agitation lorsqu'il se tient debout dans un cadre. Les signes vitaux du patient indiquent une
hypertension et une bradycardie. La réponse immédiate la plus appropriée du kinésithérapeute est la suivante :
1. apprendre au patient des techniques de respiration profonde.
2. appliquer une compresse froide cervicale.
3. retirer la ceinture abdominale.
4. évaluer l'absence d'obstruction du cathéter.

195) Lequel des schémas de facilitation neuromusculaire proprioceptive des membres inférieurs suivants est le PLUS
approprié pour un patient qui a besoin de renforcer le tibialis posterior ?
1. Extension, abduction et rotation médiale (interne) de la hanche, avec flexion plantaire et éversion de la
cheville.
2. Flexion, adduction et rotation latérale (externe) de la hanche, avec dorsiflexion et inversion de la cheville.
3. Extension, adduction et rotation latérale (externe) de la hanche, avec flexion et inversion plantaire de la
cheville.
4. Flexion, abduction et rotation interne de la hanche, avec dorsiflexion et éversion de la cheville.

196) Un kinésithérapeute évalue un patient souffrant de lombalgie et de symptômes associés. Laquelle des constatations
suivantes devrait amener le thérapeute à renvoyer le patient chez le médecin ?
1. Bonne (4/5) force des ischio-jambiers
2. Douleur irradiant l'arrière de la cuisse jusqu'au mollet
3. Test de la jambe droite positif à 60° de flexion de la hanche
4. Anesthésie en selle autour du périnée

197) Lequel des types de fibres afférentes suivants transmet l'information concernant la douleur ?
1. Grandes fibres non myélinisées
2. Petites fibres non myélinisées
3. Petites fibres myélinisées
4. Fibres myélinisées de grande taille

198) Un thérapeute mesure l'amplitude du mouvement passif du genou d'un patient. Les mesures obtenues sont
présentées sur les photographies A et B. La cause la plus probable de la différence d'amplitude de mouvement du
genou est la suivante :
1. dans la capsule de l'articulation du genou.
2. une tension dans le muscle droit du fémur.
3. faiblesse des ischio-jambiers.
4. une tension dans le vaste médial.

199) Lequel des grades monofilament suivants indique que seule la sensation de protection est intacte ?
1. No table of contents entries found.

200) Lequel des changements suivants résulte de la stimulation du système nerveux sympathique ?
1. Augmentation du flux sanguin vers peau
2. Diminution de la glycémie
3. Dilatation des bronchioles
4. Augmentation de la fréquence cardiaque

PEAT
4
27 Réponses

1. 1 Au cours de la phase I (réadaptation cardiaque en milieu hospitalier), les paramètres des signes vitaux dont
l'activité justifie la prise en charge de la réadaptation cardiaque doivent être pris en compte.
sont : une tension artérielle diastolique égale ou supérieure à 110 mm Hg, une tension artérielle systolique
supérieure à 210 mm Hg ou une augmentation de plus de 20 mm Hg par rapport au repos, et une augmentation de
la fréquence cardiaque de plus de 20 bpm par rapport au repos. La fréquence respiratoire normale au repos peut
varier de 12 à 20 respirations par minute chez l'adulte, de sorte qu'une augmentation à 20 respirations par minute
lors d'une activité de faible intensité ne constitue pas une raison de mettre fin à l'activité.

2. 3 La conduction nerveuse au-dessus et au-dessous de la compression nerveuse locale est généralement


normale. Le temps de latence est généralement de
a augmenté, et non diminué, au niveau du site de compression du canal carpien. La conduction nerveuse au-dessus
et au-dessous de la compression nerveuse locale est généralement normale.

3. 1 Avec des antécédents d'abus d'alcool et la présence de tremblements fins au repos et d'une douleur dans le
quadrant supérieur droit, le
Le patient présente des antécédents et des signes et symptômes compatibles avec une maladie du foie. La jaunisse
est une modification de la peau associée à une maladie du système hépatique. L'hyperhidrose peut être présente en
cas de troubles endocriniens, mais elle n'est pas associée aux maladies du foie. L'hypotension n'est pas mentionnée
comme un signe de troubles hépatiques. Une toux nocturne peut être associée à une fièvre rhumatismale, mais n'est
pas caractéristique d'une maladie du foie.

4. 2 La racinenerveuse C7 procure une sensation dans la partie dorsale du majeur.

5. 1 Un clonus soutenu de la cheville indique un dysfonctionnement du système nerveux central, tout comme la
présence d'un signe de Babinski.
(c'est-à-dire un gros orteil qui monte avec une poussée du pied plantaire). Les autres options sont associées à des
problèmes de motoneurones inférieurs.

6. 1 Le nourrisson présente des signes de torticolis affectant le muscle sterno-cléido-mastoïdien droit. Le


torticolis est nommé
pour le côté où la flexion latérale est limitée. L'asymétrie et la fermeture prématurée des sutures (plagiocéphalie) ne
sont pas typiques de l'hypomobilité des facettes cervicales chez les nourrissons.

7. 2 Le report de tout exercice d'amplitude n'est pas un choix pratique, car une contracture se développera
après l'opération. Étant donné que ce patient ne peut pas atteindre une amplitude de mouvement complète par lui-
même, une amplitude de mouvement active et assistée est indiquée pour prévenir la contracture postopératoire.
Bien que l'entraînement à la mobilité dans le lit soit un moyen créatif d'augmenter l'amplitude des mouvements des
membres supérieurs, compte tenu de l'acuité de la plaie chirurgicale du patient, ce dernier aurait besoin d'une plus
grande amplitude de mouvements pour que cette intervention soit plus bénéfique. Le fait de continuer à n'utiliser
que l'amplitude de mouvement active ne faciliterait pas l'augmentation adéquate de l'amplitude de mouvement et
ne préviendrait pas les contractures.

8. 2 Le courant interférentiel de niveau sensoriel de 80 à 100 Hz ne traite pas le problème lui-même mais peut
traiter
toute douleur associée. L'ionophorèse à l'acide acétique peut traiter la cause de la tendinite calcifiante, et pas
seulement les symptômes. La stimulation électrique pulsée à 200 pps ne traite pas le problème lui-même, mais peut
soulager la douleur qui y est associée. La diathermie avec une installation de traitement parallèle n'est pas le
meilleur choix, car la chaleur profonde ne traitera pas la pathologie.

9. 1 Plusieurs mesures peuvent être prises pour réduire la densité du courant et le risque d'irritation de la peau.
Ces
Il s'agit notamment de diminuer l'intensité de la stimulation, d'augmenter la distance inter-électrodes et d'utiliser
des électrodes plus grandes.

10. 1 Une note médiocre (2/5) est définie comme une excursion complète en position d'élimination de la
pesanteur. Toutes les autres options
sont contre les positions de gravité, ce qui serait inapproprié, compte tenu de ce degré de faiblesse.

11. 1 Le biceps brachial est à la fois un fléchisseur du coude et un supinateur, et c'est en tant que supinateur qu'il

PEAT
4
28 Réponses

est le plus efficace avec le


coude fléchi à environ 90° (approximativement la position de test musculaire). Le brachial ne provoque pas de
supination (seulement de la flexion). Le supinateur ne fléchit pas le coude. Le brachioradialis déplace l'avant-bras
en position médiane plutôt qu'en supination complète. Par conséquent, lorsque le coude est à la fois en flexion et en
supination, le biceps brachial est le muscle le plus susceptible d'être à l'origine de cette action.

12. 3 En ce qui concerne les options 1 et 2, pour les perturbations plus importantes, les individus utilisent les
muscles des hanches et des genoux pour récupérer.
l'équilibre. En ce qui concerne l'option 3, pour les perturbations légères, la plupart des individus utilisent une
stratégie de la cheville. La musculature de la cheville est utilisée pour contrôler la perturbation et rétablir
l'équilibre. En ce qui concerne l'option 4, si la perturbation est suffisamment forte pour que le centre de masse de
l'individu se déplace en dehors de la base d'appui, une stratégie d'enjambement serait employée en faisant un pas et
en augmentant la taille de la base d'appui.

13. 4 Par définition, le blanchiment du lit de l'ongle avec un retour de la couleur en moins de 3 secondes est un
remplissage capillaire normal.

14. 3 La position décrite crée la force nécessaire pour déplacer le bas du corps dans ce transfert, compte tenu du
niveau
de la lésion de la moelle épinière. L'extension des doigts contre résistance serait difficile pour un patient atteint
d'unetétraplégie C6 . Les mains du patient sont maintenues près de la cuisse ou des hanches, une main sur le tapis et
l'autre sur le fauteuil roulant.

15. 3 Le sphincter anal externe et les muscles du plancher pelvien sont composés de fibres musculaires striées.
Ils reçoivent
innervation somatique à partir des segments 2 à 4 de la moelle sacrée. Lorsque ces segments sont endommagés, le
sphincter et les muscles du plancher pelvien restent flasques. L'individu perd le contrôle volontaire de la
défécation. Le réflexe gastrocolique, médié par le système nerveux intrinsèque du tube digestif, revient après la
résolution du choc spinal.

16. 1 Le nerf crânien concerné est le nerf oculomoteur. Ce nerf innerve le muscle droit médian, qui, s'il est
faible, provoquerait un strabisme latéral. Le nerf oculomoteur est également responsable de la constriction
papillaire et une lésion entraînerait une dilatation papillaire. Le ptosis est causé par la perte d'innervation du muscle
levator palpabrae superioris, qui élève la paupière. La double vision serait causée par l'incapacité de bouger le
globe oculaire normalement, car quatre des six muscles oculaires sont contrôlés par le nerf oculomoteur. Le nerf
oculomoteur joue également un rôle important dans la médiation du réflexe pupillaire à la lumière. Le nerf facial
innerve les muscles de l'expression faciale. Le nerf trijumeau est le médiateur du réflexe de la mâchoire et de la
sensation de douleur au niveau du visage.

17. 4 Le staphylocoque doré résistant à la méthicilline se propageant par contact, le port d'un écran facial ou d'un
masque est recommandé.
n'est pas nécessaire. Le kinésithérapeute n'a pas besoin de précautions ou de techniques stériles pour traiter un
patient infecté par le staphylocoque doré résistant à la méthicilline.

18. 1 La meilleure position pour drainer le segment postérieur des deux lobes inférieurs est la position couchée
sur le ventre, avec l'abdomen en arrière.
la tête vers le bas et les membres inférieurs et les hanches surélevés à environ 45°.

19. 2 Les affections qui interfèrent chroniquement avec la perfusion et la nutrition des tissus peuvent être à
l'origine de l'hippocratie. Pulmonaire
La maladie d'Alzheimer est la cause la plus fréquente de l'hernie digitale, présente dans 75 à 85 % des cas. Étant
donné que les autres affections affectent également la perfusion des tissus, elles peuvent contribuer à l'affection,
mais ne figurent pas dans la liste des observations courantes.

20. 2 Les boucles du tronc sont contre-indiquées pour les patients souffrant de diastasis recti. Les élévations de
la tête en position couchée avec crochets sont accentuées.
le muscle grand droit de l'abdomen et sont les moins susceptibles d'augmenter la séparation du diastasis recti. Le
glissement des jambes par inclinaison du bassin est plus avancé que l'élévation de la tête. L'abaissement bilatéral
des jambes est un exercice de renforcement abdominal avancé qui provoque des tensions excessives dans le bas du

PEAT
4
Réponses 29

dos et ne doit pas être pratiqué pendant la grossesse.

21. 3 La famille doit être impliquée dans toutes les étapes de la planification et du traitement. L'implication de la
famille peut raccourcir
le processus de réadaptation et faciliter le retour du patient dans la communauté. Il est important d'impliquer la
famille dès le début du processus de réadaptation plutôt que d'attendre que le patient soit prêt à sortir.

22. 3 L'alignement statique pour la stabilité du genou est établi en positionnant le genou de manière à ce que la
référence latérale, c'est-à-dire l'angle d'inclinaison du genou, ne soit pas dépassé.
se situe en avant de l'articulation du genou.

23. 1 Le sartorius assure la flexion, la rotation latérale (externe) et l'abduction de l'articulation de la hanche. Lors
d'une flexion de la hanche avec résistance, le
sartorius sera recruté pour effectuer les trois actions, ce qui donnera le modèle de substitution observé. Le tenseur
du fascia latae est un rotateur et un fléchisseur médial (interne) de la hanche, de sorte que sa substitution
impliquerait une rotation et une abduction médiales (internes). Le long adducteur permet l'adduction de la hanche.
La substitution par le semimembranosus entraînerait l'extension de la hanche.

24. 1 La fiabilité inter-juges concerne la variation entre deux ou plusieurs évaluateurs qui mesurent le même
groupe d'objets.
sujets. Le coefficient de fiabilité a des valeurs comprises entre 0,00 et 1,00. Une fiabilité de 1,00 signifie qu'il y a
un accord total. Ainsi, une valeur de 0,93 signifie qu'il y a un degré élevé de concordance entre les scores du
dynamomètre et ceux de plusieurs thérapeutes. La concordance des mesures prises par le même thérapeute
constitue la fiabilité intraréférentielle.

25. 1 La variable dépendante (ROM) est le facteur causé par la variable indépendante (fréquence des
visites).

26. 4 Pour éviter toute confusion, il convient de ne présenter au patient qu'un seul nouvel élément à la fois. Le
rythme d'apprentissage
doit être fixé par le patient. L'association d'éléments visuels et auditifs peut favoriser l'apprentissage. Les exemples
concrets sont plus faciles à comprendre que les métaphores.

27. 2 En cas d'insuffisance veineuse, les membres sont œdémateux et les veines superficielles sont dilatées. Si
si l'insuffisance veineuse n'est pas corrigée, une ulcération peut se développer. Les options 1 et 3 sont exclues car
elles indiquent l'absence d'œdème. L'option 4 n'est pas correcte car l'affection est soulagée par la position assise ou
l'élévation des jambes.

28. 2 Bien que l'aspiration nasotrachéale soit une option viable pour éliminer les sécrétions d'un patient, il s'agit
généralement du dernier recours
lorsqu'un patient ne dispose pas d'une voie respiratoire artificielle. Ainsi, si l'assistance costophrénique manuelle
ne fonctionne pas, une aspiration peut s'avérer nécessaire. Tout patient ayant bénéficié d'une ventilation mécanique
présentera probablement un certain déconditionnement des muscles respiratoires. Cependant, en cas de maladie
neuromusculaire superposée, la faiblesse des muscles respiratoires sera encore plus exacerbée. Par conséquent, une
assistance manuelle au niveau des côtes inférieures pendant la toux aidera le patient à évacuer les sécrétions. Un
supplément d'oxygène peut améliorer l'endurance du muscle ventilatoire du patient, mais ne garantit pas que le
patient sera en mesure de générer une force suffisante pendant la toux. L'entraînement des muscles inspiratoires
serait bénéfique pour ce patient, mais pas à ce stade aigu, juste après l'arrêt de la ventilation. Une fois les sécrétions
maîtrisées, l'entraînement musculaire peut commencer.

29. 2 La meilleure réponse serait que le thérapeute recommande une conférence d'équipe. Cette conférence
réunir les membres de l'équipe soignante, ainsi que le patient et les membres de sa famille, afin de parvenir à une
décision commune concernant le placement du patient. Le meilleur moment pour une telle réunion serait avant la
sortie, car il est difficile d'organiser de telles réunions après la sortie. Permettre au patient de déterminer son propre
environnement de sortie peut être une bonne chose s'il n'y a pas de problèmes de sécurité ; cependant, il faut
d'abord décider si le patient est capable ou non de prendre cette décision. La programmation d'une visite à domicile
ou la mise en œuvre d'un plan de traitement pour les soins à domicile serait prématurée tant qu'une décision de
placement n'a pas été prise.

PEAT
4
30 Réponses

30. 2 L'analyse des données dans le cadre d'une recherche sur un seul sujet repose sur l'évaluation des mesures
au sein d'un même modèle et d'un modèle à l'autre
afin de déterminer si les comportements changent et si les changements observés pendant l'intervention sont
associés au début du traitement. Alors que l'analyse visuelle de la représentation graphique des données est la
méthode d'analyse des données la plus couramment utilisée dans les études à sujet unique, l'analyse statistique
fournit une approche plus quantitative pour déterminer si les changements observés sont réels ou s'ils sont le fruit
du hasard.

31. 2 Les tissus macérés résultent d'une humidité excessive. Un pansement plus absorbant absorberait le
l'humidité excessive et empêcher la macération.

32. 3 Trois semaines, c'est trop tôt pour soulever des objets de 2,3 kg. Trois semaines, c'est trop tôt pour
s'endormir sur l'engagement
côté. En général, au bout de 8 à 12 semaines, un patient qui a subi une réparation ouverte sans complication de la
coiffe des rotateurs il y a deux semaines est capable d'élever activement son bras à des hauteurs fonctionnelles.
Trois mois, c'est trop tôt pour jouer au golf.

33. 3 Étant donné que le patient ne porte pas de poids, les capacités de transfert et l'examen de la démarche sont
moins susceptibles de donner des résultats.
des informations sur la cause de la douleur au pied. En ce qui concerne le remplissage capillaire des orteils, les
complications de la fixation par plâtre peuvent inclure un gonflement et, en cas de gravité, un syndrome des loges.
Des plâtres mal adaptés et/ou des patients laissant leurs membres en position de dépendance peuvent entraîner un
gonflement douloureux qui crée une occlusion du flux sanguin distal. Le test de remplissage capillaire est un test
facile pour examiner la perfusion des extrémités distales. Les variations de la pression artérielle peuvent entraîner
des modifications de la perfusion des extrémités distales, mais il serait très difficile de mesurer la pression
artérielle des extrémités inférieures lorsque le patient est plâtré.

34. 1 Le schéma capsulaire de restriction à la hanche est une perte de rotation médiale (interne) et d'abduction,
suivie d'une perte d'amplitude.
par une perte de la flexion et de l'extension ; la perte de la rotation latérale (externe) est insignifiante.

35. 2 Les options 1, 3 et 4 ne fournissent pas un soutien orthétique suffisant pour permettre à l'enfant de
marcher. Un enfant avec uneL1
ne peut marcher qu'avec l'appui d'une orthèse de marche à mouvement alternatif ou d'une orthèse thoraco-lombo-
sacrée.

36. 2 La prednisone est un glucocorticoïde qui présente les effets secondaires décrits dans la tige. L'effet
secondaire principal
de la pénicilline est une réaction allergique, telle que des éruptions cutanées et des difficultés respiratoires. Le
principal effet secondaire de l'aspirine est la gastrite. Le principal effet secondaire de la thérapie à l'or est la
diarrhée, l'irritation de la muqueuse buccale et les éruptions cutanées.

37. 2 L'exercice physique contribue grandement au contrôle de l'hyperglycémie dans le diabète de type 2 en
améliorant la santé du squelette.
le transport du glucose dans le muscle et l'homéostasie du glucose dans l'ensemble du corps. L'exercice régulier
peut aider l'organisme à répondre à l'insuline et est connu pour son efficacité dans la gestion de la glycémie.
L'exercice physique peut réduire la glycémie et éventuellement diminuer la quantité de médicaments nécessaires
pour traiter le diabète, voire éliminer le besoin de médicaments. Il n'a pas été démontré que l'exercice physique
améliore le contrôle de la glycémie chez les personnes atteintes de diabète de type 1. Les patients atteints de
maladies neuromusculaires dégénératives telles que la sclérose en plaques doivent faire preuve de prudence
lorsqu'ils font de l'exercice afin d'éviter une fatigue excessive, qui peut entraîner des pertes de force permanentes.
L'atteinte du système respiratoire chez les personnes atteintes de sclérose en plaques et de sclérose latérale
amyotrophique peut entraîner une mauvaise tolérance à l'exercice aérobique intense.

38. 4 Il n'est pas conseillé d'utiliser de la gaze saturée de solution saline pour contrôler les écoulements
abondants. Le film semi-perméable ne peut pas
absorbent de grandes quantités d'exsudat. La gaze imprégnée d'oxyde de zinc ne s'est pas révélée bénéfique. La
pâte hydrocolloïde est la seule option proposée qui conviendrait à la prise en charge des plaies présentant des
niveaux élevés d'exsudat.

PEAT
4
Réponses 31

39. 2 Une flexion plantaire prothétique excessive peut entraîner une flexion insuffisante du genou. Un coussin
de talon rigide peut causer
une flexion excessive du genou et donc une déformation. Un talon de chaussure prothétique bas peut entraîner un
retard de flexion du genou. Un enfoncement excessif du pied prothétique peut entraîner une poussée latérale
excessive.

40. 4 Les exercices qui nécessitent une position couchée doivent être effectués avant le repas afin que l'estomac
soit bien calé.
relativement vide. La durée recommandée pour rester debout après un repas est de 3 heures. Le soulèvement de la
tête en décubitus dorsal est l'exercice recommandé pour renforcer le muscle du sphincter supérieur de l'œsophage.
Si l'exercice est effectué en position verticale, la tête tombe sous l'effet de la gravité et l'exercice n'est plus
résistant. Le patient doit se coucher sur le côté gauche pour réduire le reflux, car le fait de se coucher sur le côté
droit facilite l'écoulement de l'acide dans l'œsophage, étant donné que la partie inférieure de l'œsophage se plie vers
la gauche et se redresse en se couchant sur le côté droit.

41. 1 La cyanose est décrite dans la tige. Bien que cette constatation accompagne souvent des troubles
cardiaques/pulmonaires, il n'est pas rare que les patients ne soient pas satisfaits de leur état de santé.
ou peut survenir en cas de troubles hématologiques ou du système nerveux central, la description objective est la
cyanose.

42. 2 Les dysfonctionnements de l'intestin et de la vessie sont liés à un dysfonctionnement de la moelle épinière
et du motoneurone supérieur. Fumer et
Les stéroïdes peuvent affaiblir les os et augmenter la probabilité d'une fracture par compression. L'hypertension et
le diabète ne sont pas des facteurs de risque de fracture par compression. En soi, ni l'emphysème ni le traitement
hormonal substitutif n'augmentent la probabilité d'une fracture par compression.

43. 4 Les bains de cire de paraffine sont utilisés dans les phases non éruptives pour diminuer la douleur et
augmenter l'extensibilité des tissus dans les cas suivants
patients atteints de polyarthrite rhumatoïde. L'augmentation de l'extensibilité du collagène associée à
l'échauffement peut diminuer la perception de la douleur et augmenter la tolérance et la participation à l'exercice
actif. L'option 1 est incorrecte car la cyrothérapie (poche de gel froid) diminue l'extensibilité des tissus. Les options
2 et 3 sont incorrectes car ces modalités ne sont pas faciles à appliquer sur des zones où les tissus mous sont fins
(mains et doigts). La paraffine permet une répartition uniforme de la chaleur sur les doigts (toutes les articulations).

44. 3 L'extension du tronc est l'exercice le plus sûr pour les patients souffrant d'ostéoporose et est nécessaire,
compte tenu du risque élevé d'ostéoporose.
pour la cyphose avec fractures vertébrales. Les autres mouvements du tronc sont contre-indiqués pour les patients
souffrant d'ostéoporose.

45. 4 La fièvre peut s'accompagner de modifications du rythme cardiaque, de la pression artérielle ou de la


fréquence respiratoire.
Ces mesures permettront de déterminer la cause de la fièvre. Les causes courantes de fièvres postopératoires
immédiates sont l'atélectasie ou la pneumonie. L'auscultation des poumons aiderait le thérapeute à déterminer la
cause de la fièvre et à définir l'intervention physiothérapeutique appropriée.

46. 4 Afin d'intervenir au mieux auprès d'un patient souffrant d'un dysfonctionnement cardiaque, il convient de
procéder à un examen complet et à une évaluation de l'état de santé du patient.
sont nécessaires à l'élaboration d'un plan de traitement adéquat. Un aspect important de l'examen consiste à
déterminer le type d'angine dont souffre le patient afin que le thérapeute sache comment prévenir l'angine par
l'exercice ou la reconnaître si elle survient au cours de la séance de traitement. Les autres options proposées sont
toutes des résultats qui se produiraient après que le patient a terminé un programme de réadaptation cardiaque.

47. 2 Contacter l'infirmière à domicile du patient est la première mesure à prendre en raison de la possibilité de
le risque de sécurité associé au fait que le patient ne prenne pas d'insuline. Les injections et les tests de glycémie
dépassent le champ d'application de la kinésithérapie. Faire porter la responsabilité à la famille ne serait pas
approprié en raison de la gravité de la situation.

48. 2 Les objectifs doivent être mesurables et le comportement spécifique attendu doit être précisé. Comprendre,
La liste des cinq techniques est une activité qui peut être documentée et qui est donc mesurable.

PEAT
4
32 Réponses

49. 3 Les ischio-jambiers sont des extenseurs de la hanche qui sont plus actifs lorsque la hanche est en flexion,
en particulier dans les situations fonctionnelles.
des activités telles que la montée d'escaliers. Dans ce cas, le patient fléchit la hanche, ce qui met le demi-tendineux
en tension et augmente son bras de moment ainsi que sa capacité à produire une extension de la hanche. Par
conséquent, le patient se penche en avant pour maximiser la capacité des ischio-jambiers à étendre la hanche
pendant la montée de l'escalier. Les autres muscles énumérés ne bénéficieraient pas autant de l'augmentation de la
flexion de la hanche.

50. 3 Un échantillon de petite taille peut être lié à des erreurs dans les conclusions statistiques parce qu'il n'y a
peut-être pas assez d'échantillons.
participants pour permettre l'application des résultats à tous les patients de la population concernée. La variance à
l'intérieur du groupe étant plus faible, les distributions des échantillons se chevauchent moins. Une puissance
d'analyse supérieure à 80 % est liée à une plus grande probabilité que les conclusions de l'étude soient exactes. Un
niveau alpha élevé (c'est-à-dire non contrôlé) se traduit par un niveau de signification statistique plus élevé et une
plus grande probabilité d'erreur de type 1.

51. 3 Au fur et à mesure que l'apprentissage progresse, le retour d'information doit passer de l'extrinsèque
(tactile, visuel, verbal) à l'extrinsèque (tactile, visuel, verbal).
rétroaction intrinsèque du fuseau musculaire et des récepteurs articulaires.

52. 2 Le complexe QRS représente la contraction ventriculaire. La contraction auriculaire est représentée par
l'onde P.
Lorsqu'une zone du ventricule devient irritable et développe un foyer ectopique, le ventricule se dépolarise
prématurément avant la séquence de conduction normale (par exemple, avant le déclenchement du nœud SA dans
les oreillettes) et se présente sous la forme d'un complexe QRS large et irrégulièrement espacé. En cas de
dépolarisation ventriculaire normale, le QRS est étroit et régulièrement espacé et la fibrillation auriculaire apparaît
lorsque l'onde P se modifie. La repolarisation auriculaire se produit dans le complexe QRS et n'est normalement
pas visible sur l'électrocardiogramme.

53. 1 La surélévation et la pression sont les mesures correctes à prendre pour contrôler le saignement. Retrait du
pansement et
risquerait d'irriter la plaie et d'aggraver l'hémorragie. L'élévation et la pression doivent être appliquées en premier,
avant les pansements, bandages et pressions supplémentaires sur l'artère brachiale. S'il peut être nécessaire
d'appeler le médecin du patient pour organiser le transport vers des soins médicaux, cela ne permet pas de
contrôler immédiatement l'hémorragie.

54. 4 Le thérapeute recherche un signe de Babinski positif. Lorsque le signe de Babinski est positif, les orteils
s'évasent à
la fin du test. La présence d'un signe de Babinski positif indique une lésion du motoneurone supérieur, comme une
lésion de la moelle épinière. En l'absence de lésion du motoneurone supérieur, les orteils fléchissent. La seule
lésion du motoneurone supérieur est une lésion de la moelle épinière. Les lésions des nerfs périphériques et des
cellules de la corne antérieure sont des lésions des motoneurones inférieurs.

55. 4 La photo montre un bureau debout/un poste de travail. Un patient souffrant d'une sténose se portera mieux
en position assise, et non en position couchée.
debout. Un patient souffrant d'une thrombose veineuse profonde devra bouger et non rester immobile. Une scoliose
ne justifie pas à elle seule un poste de travail debout. La position assise augmente la pression intradiscale, c'est
pourquoi la position debout est souvent préférée à la position assise.

56. 3 L'artère tibiale postérieure est la plus susceptible d'être impliquée dans le syndrome chronique du
compartiment postérieur,
qui est décrite dans la question. Cette artère doit être palpée en arrière de la malléole médiale. La face dorsale du
pied est le site de palpation du pouls du pédis dorsal. L'artère dorsale du pied n'est pas impliquée dans le syndrome
du compartiment postérieur. Le site de palpation postérieur à la malléole latérale est utilisé pour accéder à l'artère
fibulaire (péronière). L'artère fibulaire (péronière) n'est pas impliquée dans le syndrome du compartiment
postérieur. La palpation du creux poplité est un mauvais choix de localisation de la palpation en association avec le
syndrome du compartiment postérieur. L'artère poplitée peut être palpée à cet endroit, mais les manifestations du
syndrome du compartiment sont distales par rapport à ce site.

PEAT
4
Réponses 33

57. 1 Il a été établi qu'un facteur essentiel pour améliorer l'apprentissage est que le sujet doit faire quelque chose
différents lors d'essais consécutifs. Par conséquent, l'approche traditionnelle du recyclage par la pratique répétée
d'une compétence n'est pas la plus efficace. On pense que le fait de demander à un patient de s'exercer à un certain
nombre de tâches dans un ordre aléatoire serait probablement plus efficace pour la rétention à long terme. Les
autres options ne prévoient pas de randomisation des activités et sont donc moins susceptibles d'assurer une
rétention à long terme.

58. 1 Les muscles abdominaux sont actifs pendant une position assise (genoux tendus) jusqu'à ce que la colonne
vertébrale soit en contact avec le sol.
complètement fléchi (tête, épaules, thorax soulevés de la surface). Or, pour se mettre en position assise longue, il
faut fléchir les hanches, ce que les abdominaux ne peuvent pas faire puisqu'ils ne traversent pas l'articulation de la
hanche. Par conséquent, les fléchisseurs de la hanche (iliopsoas entre autres) doivent effectuer ce mouvement.
L'incapacité à adopter une position assise prolongée suggère une faiblesse du muscle iliopsoas.

59. 3 Les myopathies induites par les corticostéroïdes sont fréquentes après une utilisation prolongée du
médicament, et elles sont fréquentes dans les cas suivants
la musculature proximale des extrémités. Les autres options sont à l'opposé des symptômes que l'on retrouve
généralement dans les pathologies induites par les stéroïdes. En général, il y a des problèmes d'hypertension,
d'hypocaliémie et d'augmentation du métabolisme des protéines.

60. 1 Un patient immobile et alité risque de développer une atélectasie (affaissement partiel de l'estomac).
Le cancer du poumon est une maladie grave qui peut entraîner une pneumonie. Des changements de position
fréquents avec une respiration et une toux profondes aideront à prévenir le développement de l'atélectasie. Étant
donné que ce patient est âgé et qu'il n'y a pas de diagnostic de rétention de sécrétions, une percussion et une
vibration vigoureuses ne sont pas indiquées. Les vibrations avec la tête en bas ou les positions de drainage postural
standard ne seront pas tolérées par ce patient âgé souffrant d'insuffisance cardiaque congestive chronique.

61. 4 L'extension de la première articulation métatarso-phalangienne est nécessaire en position terminale. Le


métatarsien distal est
convexe. La partie proximale de la phalange articulée est concave. Selon la règle convexe-concave, lorsqu'un
thérapeute déplace une surface articulaire concave sur une surface articulaire convexe, la surface articulaire
concave est déplacée dans la même direction que la limitation de l'amplitude de mouvement. Par conséquent, le
glissement accessoire approprié est un glissement dorsal de la phalange proximale sur le métatarsien. Le
glissement postérieur de l'astragale sur le tibia et le glissement plantaire de la 1ère phalange proximale sur le
métatarsien favorisent la flexion plantaire. Le glissement postérieur du calcanéum sur l'astragale n'est pas aussi
important pour l'appui terminal, qui implique la première articulation MTP.

62. 1 Le fait que le patient présente une force musculaire normale des rotateurs latéraux (externes) de la hanche
ainsi que des
la limitation de l'amplitude des mouvements indique soit une tension des rotateurs médians (internes), soit une
tension de la capsule de la hanche. Le moyen et le petit fessier sont des rotateurs internes de la hanche. Le
déplacement du tronc est un schéma de substitution normal que l'on retrouve chez les personnes dont l'amplitude
de mouvement des hanches est insuffisante. Le problème est lié à la hanche et non au tronc. Le piriforme du patient
présente une force normale, il n'est donc pas faible.

63. 2 Un patient souffrant d'une lésion de la moelle épinière thoracique est capable de mettre en œuvre des
stratégies indépendantes de soulagement de la pression.
et doit procéder à une décompression toutes les 15 à 20 minutes.

64. 4 Une mauvaise croissance des cheveux est caractéristique d'une maladie artérielle due à une nutrition
cellulaire inadéquate. Le premier signe de
La maladie artérielle se traduit souvent par une perte de poils sur les orteils. Un œdème peut être présent dans les
cas avancés de maladie artérielle. Une cyanose accrue avec les jambes en position de dépendance est présente à un
stade avancé de la maladie. La décoloration brunâtre des chevilles est caractéristique des troubles veineux
chroniques.

65. 3 Les options 1, 2 et 4 ne sont ni sûres ni appropriées. Lors de la descente d'un escalier, il est généralement
préférable que le
thérapeute à placer sous le patient pour le protéger contre les chutes. Cependant, le patient doit toujours descendre

PEAT
4
34 Réponses

les escaliers en commençant par la jambe concernée, la jambe droite dans ce cas, et la canne.

66. 3 Le cancer du pancréas se caractérise par des symptômes non spécifiques et vagues, qui peuvent inclure des
nausées, une perte de poids, des douleurs abdominales et des douleurs musculaires.
douleur irradiant vers le dos ou douleur dorsale seule, et jaunisse. Le fait de s'asseoir et de se pencher en avant peut
apporter un certain soulagement en réduisant la pression exercée sur le pancréas. Le syndrome du côlon irritable se
caractérise par des douleurs abdominales accompagnées de constipation et de diarrhée, de nausées et de
vomissements, mais pas de douleurs dorsales ni de jaunisse. La cholécystite est une inflammation de la vésicule
biliaire qui provoque généralement des douleurs abdominales dans le quadrant supérieur droit. Il peut y avoir des
nausées et des vomissements, une perte de poids et une jaunisse. Cependant, la douleur est dirigée vers le haut du
dos ou l'épaule droite, et non vers le bas du dos. L'appendicite s'accompagne également d'anorexie, de nausées et
de vomissements, mais la douleur est généralement présente dans le quadrant inférieur droit. Se pencher en avant
peut aggraver les symptômes en raison de l'augmentation de la pression intra-abdominale qui en résulte.

67. 4 Une rotation excessive de l'omoplate vers le haut peut résulter d'une faiblesse des rhomboïdes et du
latissimus dorsi.
(rotateurs vers le bas). L'adduction de l'omoplate avec rotation médiale (interne) et adduction du bras nécessiterait
l'action de ces muscles. L'option 1 permet de renforcer le serratus anterior, un rotateur de l'omoplate vers le haut.
L'option 2 activerait le trapèze supérieur ainsi que les rhomboïdes et, comme le trapèze supérieur est également un
rotateur vers le haut de l'omoplate, ce ne serait pas le meilleur exercice à utiliser. L'option 3 permettrait également
de renforcer le serratus anterior, ce qui aurait tendance à aggraver le problème.

68. 4 L'hyperplasie nodulaire qui se produit dans le cas de l'hyperplasie bénigne de la prostate provoque une
obstruction de l'urètre,
entraînant une fréquence urinaire et une incontinence par impériosité. La torsion testiculaire, l'orchite et le cancer
du testicule ne sont pas associés à un dysfonctionnement urinaire.

69. 1 Le transfert en squat-pivot permet d'éviter l'extension complète des membres inférieurs, ce qui minimise
l'action des extenseurs des membres inférieurs.
ton. Un transfert sur une planche coulissante n'est pas approprié en cas d'ataxie importante des membres supérieurs.
Un transfert deboutn'est pas approprié car il peut augmenter le tonus des extenseurs des membres inférieurs. Le
transfert d'une personne dépendante n'encourage pas la participation active du patient et ne favorise pas son
indépendance.

70. 4 La question de recherche porte sur la relation ou la corrélation entre les mesures, et non sur la comparaison
des groupes.
Le test t et le test U de Mann-Whitney ne sont donc pas appropriés. Les données des deux variables étant classées
(ordinales), il convient d'utiliser le rho de Spearman (coefficient de corrélation de rang de Spearman). Il s'agit de
l'analogue non paramétrique du coefficient de corrélation de Pearson (r). Le r de Pearson nécessite des données
continues et non ordinales.

71. 1 Un coefficient de corrélation compris entre 0,26 et 0,49 est considéré comme faible.

72. 1 La cage thoracique ne peut pas s'étendre normalement pendant l'inspiration en raison d'une faiblesse de la
partie externe de la cage thoracique.
les muscles intercostaux, qui sont innervés par des segments du nerf thoracique. En cas de lésion de la moelle
épinière auniveau C5-C6, le diaphragme reste innervé par le nerf phrénique (C4). Le scalène antérieur (C4-C6) serait
partiellement innervé et le sterno-cléido-mastoïdien (C2-C3) serait entièrement innervé. Les muscles abdominaux
ne seraient pas innervés puisqu'ils reçoivent leur innervation des segments du nerf thoracique. La paralysie des
muscles abdominaux entraînerait une position de repos basse du diaphragme.

73. 1 L'emplacement de l'électrode sur la photographie est le meilleur emplacement pour le tenseur du fasciae
latae. Les
Les électrodes sont placées sur le ventre du muscle et alignées parallèlement aux fibres musculaires. Le sartorius se
trouve plus distalement et médialement et suit la direction des fibres musculaires. Le rectus femoris se trouve plus
distalement et au centre de la cuisse. Le petit fessier est un muscle profond qui ne peut pas être facilement contrôlé
directement par électromyographie de surface.

74. 2 L'augmentation de la force musculaire est due à un certain nombre de facteurs, notamment l'adaptation

PEAT
4
Réponses 35

neurologique et l'augmentation de la force musculaire.


hypertrophie des fibres avec augmentation de l'actine et de la myosine. L'hyperplasie chez l'homme est encore
controversée. Les changements à long terme de la force musculaire sont dus à tous les facteurs mentionnés.
Cependant, les changements à court terme, tels que ceux observés en une semaine, sont très probablement dus à
des facteurs neurologiques tels qu'un recrutement plus efficace des unités motrices, une inhibition autogène et une
coactivation plus efficace des groupes musculaires.

75. 4 Bien que toutes les options soient impliquées dans le processus de prise de décision, la détermination de
l'option la plus appropriée est celle de l'option la plus appropriée.
Les contre-indications sont la première chose à considérer, car toutes les autres options sont inutiles si le patient
présente d'autres pathologies susceptibles de constituer une contre-indication.

76. 2 Une faiblesse du pectoral mineur n'entraînerait pas de restriction de l'omoplate, mais provoquerait
probablement
hypermobilité scapulaire. Une faiblesse du trapèze supérieur diminuerait la rotation vers le haut de l'omoplate lors
de la flexion et de l'abduction de l'épaule. Plus l'épaule est surélevée, plus ce phénomène est perceptible. La
diminution du mouvement de l'omoplate augmenterait la prédisposition à l'impaction. Une faiblesse du deltoïde
entraînerait un mouvement de l'humérus vers le bas, et non vers le haut, lors de l'élévation de l'épaule. Une
faiblesse du grand rhomboïde n'entraînerait pas de restriction de l'omoplate, mais provoquerait probablement une
hypermobilité de l'omoplate.

77. 3 Le personnel des services médico-sociaux s'occupe des situations familiales et des aides financières et joue
le rôle de directeur de ressources.
au nom du patient. Les services sociaux seraient le choix le plus approprié pour aider à organiser et à coordonner
les services de réadaptation pour le patient pendant qu'il est à domicile. Bien que les infirmières, les
ergothérapeutes, les kinésithérapeutes et les médecins puissent tous être impliqués dans les soins directs aux
patients, ils ne seraient pas les plus appropriés pour ce niveau de planification de la sortie.

78. 3 La fiabilité intraréférentielle décrit la répétabilité des mesures effectuées par une personne (répétabilité au
sein d'une même personne).
cette personne).

79. 4 En ce qui concerne la lombalgie chronique d'origine somatique, le patient souffre d'un problème
chronique, et il est donc probable qu'il va
nécessitent une stimulation motrice, car elle procure une analgésie de plus longue durée. Rien n'indique que la
stimulation sensorielle soit efficace dans le traitement de l'œdème. Pour la réduction de l'œdème, la contraction
rythmique des muscles est préférable. L'ionophorèse est plus appropriée que la stimulation électrique sensorielle
pour le traitement d'une tendinite, car elle implique l'administration de médicaments anti-inflammatoires. Le
syndrome de Raynaud est une affection dans laquelle les plus petites artères qui amènent le sang aux doigts ou aux
orteils se contractent en cas d'exposition au froid ou de choc émotionnel. La stimulation sensorielle des racines et
des troncs nerveux peut augmenter la vasodilatation périphérique.

80. 3 La maladie vasculaire périphérique est un problème vasculaire, pas un problème pulmonaire.
L'insuffisance cardiaque congestive peut
entraînent des problèmes pulmonaires, mais pas des problèmes obstructifs. L'emphysème est une maladie
pulmonaire obstructive, pour laquelle la respiration sur les lèvres peut être bénéfique. La sarcoïdose est une
maladie pulmonaire restrictive pour laquelle la respiration par les lèvres n'est pas bénéfique.

81. 3 Les antécédents suggèrent qu'un mauvais positionnement prolongé de la colonne cervicale a entraîné des
douleurs cervicales. A
la tête chronique en avant et la cyphose entraînent une hyperextension de la colonne cervicale supérieure et une
flexion excessive de la colonne thoracique supérieure. Une adaptation supplémentaire de la longueur des muscles
se produit lorsque les muscles antérieurs sont tendus et les muscles postérieurs étirés. Le traitement doit
comprendre la correction de la faiblesse ou du déséquilibre musculaire. Le renforcement des rhomboïdes et
l'extension axiale du cou sont les seules options correctes.

82. 4 L'hypoglycémie se manifeste souvent par des troubles de l'élocution et de la concentration, alors que
l'hypoglycémie se manifeste souvent par des troubles de l'élocution et de la concentration.
en cas d'hyperglycémie, la confusion est totale. Les autres options sont toutes des signes d'hyperglycémie et non

PEAT
4
36 Réponses

des signes d'hypoglycémie.

83. 4 Ni le glissement postérieur, ni le glissement antérieur, ni le glissement supérieur n'améliorent l'abduction.


La radiographie
montre une limitation du mouvement gléno-huméral à l'origine de la réduction de l'abduction de l'épaule. Le
glissement inférieur permet d'améliorer l'abduction.

84. 1 Le réapprovisionnement des cabines de traitement en fournitures est la seule option qui n'implique pas
l'intervention directe des patients.
Il s'agit donc de l'activité la plus appropriée à déléguer à un bénévole. Bien que les volontaires puissent parfois être
impliqués dans des activités de soins aux patients (par exemple, le transport de patients), les patients dans les
situations décrites dans les options 2, 3 et 4 sont exposés à un risque potentiel et nécessiteraient d'être supervisés
par une personne autre qu'un volontaire.

85. 4 Il est peu probable qu'un patient souffrant d'hémiplégie chronique récupère une fonction normale. Les
stratégies compensatoires sont
utilisé lorsqu'il y a une perte permanente de la fonction qui empêche de retrouver des schémas de mouvement
normaux. L'accoutumance est une diminution de la réactivité qui se produit à la suite d'une exposition répétée à un
stimulus non douloureux. Chez le patient en phase aiguë, l'accent est mis sur le rétablissement d'une fonction
normale, mais ce patient est atteint d'une maladie chronique. La sensibilisation est une réactivité accrue à la suite
d'un stimulus menaçant ou nocif.

86. 2 En effet, l'extension du genou en position assise et l'élévation de la jambe droite aboutissent à des positions
essentiellement identiques,
les réponses symptomatiques aux deux types de manœuvres devraient être similaires. Si le patient présentait une
hernie discale symptomatique, les deux positions entraîneraient une augmentation similaire des symptômes. Si le
patient souffrait d'une irritation du nerf sciatique, les deux positions entraîneraient une augmentation similaire des
symptômes. Si le patient avait un ischio-jambier facilité, les deux positions entraîneraient une réponse
symptomatique similaire.

87. 2 La largeur du siège doit être légèrement supérieure à la largeur de la partie la plus large du corps, et la
profondeur doit venir de la partie la plus large du corps.
à moins d'un pouce du creux poplité.

88. 4 En ce qui concerne l'hypertension du patient, le contraire est probable. L'hypotension orthostatique est une
effet secondaire. L'arythmie ne figure pas parmi les effets secondaires reconnus des opioïdes. En ce qui concerne la
diarrhée, la motilité gastro-intestinale est diminuée, de sorte que l'effet inverse de la constipation est un problème
fréquent. Les opioïdes ont tendance à rendre les chimiorécepteurs médullaires moins sensibles au dioxyde de
carbone, ralentissant ainsi la fréquence respiratoire et induisant une hypoxie et une hypercapnie relatives. La
réponse respiratoire à l'exercice peut être émoussée.

89. 4 Les gants ne sont nécessaires qu'en cas de contact avec du sang ou des fluides corporels, et non lors des
soins aux patients qui ne sont pas
impliquent un contact avec du sang ou des fluides corporels. Des gants sont nécessaires pour changer les couches
d'un nourrisson, car cette activité implique un contact avec des fluides corporels.

90. 2 La structure indiquée par la flèche est le tubercule supérieur. Le bras est en rotation médiale (interne) dans
cette position.
radiographie.

91. 1 Une réponse normale est 2 ou plus (+). Un moins (-) ou 0 indique un réflexe absent, et un moins (-) ou 1
indique un réflexe absent.
une diminution de la réponse. Plus (++) ou 3 indique une réponse exagérée, et plus (+++) ou 4 indique un clonus.

92. 4 La présence d'un clonus se situerait du côté droit, et non du côté gauche. La spasticité se situe du côté
droit,
pas le côté gauche. Il s'agit d'un cas d'hémisection de la moelle épinière, le syndrome de Brown-Séquard. La lésion
se situe sur le côté droit de la moelle épinière et il s'agit d'une lésion du motoneurone supérieur (signe de Babinski
positif) avec atteinte du tractus corticospinal, des colonnes postérieures et du tractus spinothalamique latéral. Les

PEAT
4
Réponses 37

lésions du motoneurone supérieur de la moelle épinière se manifestent par des réflexes tendineux hyperactifs (dans
ce cas du côté droit), un clonus et une spasticité ipsilatérale au côté de la lésion. La sensation de pression est perdue
du côté ipsilatéral, et la douleur et la température sont perdues du côté controlatéral à la lésion, en raison du
croisement de ces fibres dans la moelle épinière.

93. 4 L'option 1 ne s'applique qu'après que le thérapeute a procédé à une analyse plus approfondie de la
situation. Contrôle
La saturation en oxygène est utile, mais ce n'est pas la chose la plus immédiate à surveiller. Le thérapeute doit
surveiller l'électrocardiogramme et la tension artérielle du patient. Bien que le thérapeute doive avertir l'infirmière,
il doit le faire après une première évaluation de la stabilité des contractions ventriculaires prématurées. L'apparition
de contractions ventriculaires prématurées peut être bénigne ou stable. Moins de six contractions/minute est
généralement stable, tandis que plus de six contractions/minute est considéré comme moins stable. Un
kinésithérapeute devrait être en mesure de déterminer cette stabilité.

94. 4 L'option 1 est incorrecte car la position traditionnelle pour drainer le lobe moyen droit est la position
latérale gauche, et non la position latérale droite.
la mise à l'écart de la droite. L'option 2 est la position de drainage traditionnelle pour le lobe moyen droit, mais elle
ne tient pas compte des problèmes de pression intracrânienne. L'option 3 est la position de drainage traditionnelle
pour le lobe moyen droit, mais elle ne tient pas compte des problèmes de pression intracrânienne. La position
traditionnelle pour drainer le lobe moyen droit est la position latérale gauche avec une rotation d'un quart vers le
décubitus dorsal, la tête du lit étant abaissée d'environ 20°. Cependant, en raison de la craniotomie récente,
l'augmentation de la pression intracrânienne du patient est un problème majeur à surveiller et à maintenir stable.
Par conséquent, cette position traditionnelle devrait être modifiée pour devenir une position latérale à plat, comme
décrit dans l'option 4.

95. 4 Les objectifs comportementaux doivent être centrés sur l'apprenant, orientés vers les résultats, spécifiques
et mesurables. L'option 4 est
le seul qui soit centré sur l'apprenant (étudiant) et spécifique à une situation. Les options 1 et 3 requièrent une
action de la part de l'instructeur clinique, et non de l'étudiant. L'option 2, qui peut constituer une étape du
processus, n'est pas aussi centrée sur l'apprenant ou sur les résultats que l'option 4.

96. 2 Un conflit entre les racines nerveuses provoque généralement des douleurs radiculaires. L'une des causes
les plus fréquentes d'un
dans l'épaule est une arthrite traumatique consécutive à une lésion de l'épaule. La forme capsulaire peut se
développer progressivement au fil du temps. Un arc douloureux est souvent associé à un modèle non capsulaire,
comme dans le cas d'une bursite ou d'une tendinite à l'épaule. Une déchirure de la coiffe des rotateurs est associée à
une faiblesse et à une perte d'élévation active de l'épaule.

97. 3 La largeur d'impulsion de 100 microsecondes rend difficile l'obtention d'une réponse suffisamment forte
du moteur. A 1 to
La fréquence de 4 pps peut entraîner une réaction motrice de torsion, mais pas de tétanie, qui est nécessaire pour le
renforcement. La bonne réponse exige une fréquence qui entraîne une tétanie (supérieure à environ 30 Hz) et une
durée d'impulsion suffisamment longue pour recruter des fibres motrices (généralement supérieure à 200
microsecondes). Par conséquent, parmi les options possibles, 40 à 50 pps avec une largeur d'impulsion de 350
microsecondes est la meilleure. L'option 4 décrit le paramètre typique de la stimulation nerveuse électrique
transcutanée conventionnelle pour le traitement de la douleur aiguë. Cependant, la largeur d'impulsion de 100
microsecondes rend difficile l'obtention d'une réponse motrice, et la fréquence plus élevée de 100 pps entraîne une
fatigue rapide.

98. 3 Le test représenté sur la photographie est le test de Froment. Les deux pouces ( 1er chiffre) doivent rester
étendus.
pendant le test. Si le pouce (1er doigt) fléchit, cela indique une faiblesse de l'adducteur pollicien remplacée par le
fléchisseur pollicien long, ce qui est généralement dû à une lésion du nerf ulnaire.

99. 4 La position couchée avec le genou tendu serait utilisée pour tester la note médiocre (2/5). Le décubitus
dorsal avec le genou plié est utilisé pour
isoler le grand fessier. La flexion de la hanche en position assise est utilisée pour tester la force musculaire des
fléchisseurs de la hanche. La cause la plus probable de la flexion latérale vers le membre d'appui est une faiblesse
des abducteurs du côté d'appui. La flexion latérale permet de compenser la faiblesse des abducteurs. Les

PEAT
4
38 Réponses

abducteurs droits seront testés en position latérale gauche pour une force moyenne (3/5) ou supérieure.

100. 2 Bien que le test de Romberg permette de mesurer le rôle de la vision dans l'équilibre, il ne constitue pas à
lui seul la méthode la plus efficace pour mesurer l'équilibre.
mesure appropriée de l'équilibre fonctionnel. L'échelle d'équilibre de Berg est une mesure objective des capacités
d'équilibre statique et dynamique et consiste en 14 tâches fonctionnelles couramment effectuées ; c'est donc l'outil
le plus approprié pour mesurer l'efficacité de l'intervention. L'évaluation de Fugl-Meyer est appropriée pour les
AVC corticaux et ne serait pas la plus appropriée pour un AVC cérébelleux. Elle n'est pas aussi complète que
l'échelle d'équilibre de Berg en ce qui concerne les tâches d'équilibre. L'indice de Barthel est un instrument plus
global et n'est pas aussi axé sur l'équilibre fonctionnel que l'échelle d'équilibre de Berg.

101. 4 Une compression du nerf médian (syndrome du canal carpien) se produit, probablement à la suite de
l'enflure associée au fait que l'individu est dans le huitième mois de grossesse. Dans ce cas, le repos et l'élévation
sont les mesures les plus efficaces pour réduire l'œdème et soulager les symptômes. Le poignet ne doit pas être
positionné en extension complète. Le traitement conservateur initial comprend parfois la mise en place d'une attelle
pour maintenir le poignet en position neutre jusqu'à 10° d'extension, mais pas en extension complète. Bien que des
exercices de glissement du tendon puissent être utilisés, la chaleur n'est pas indiquée car elle peut augmenter
l'œdème. Les exercices de résistance pour le poignet et les doigts peuvent aggraver la compression dans le canal
carpien.

102. 2 La racinenerveuse L3 sort entre les troisième et quatrième vertèbres lombaires et fournit des informations
sensorielles.
à partir de la région de la ligne médiane de l'articulation du genou.

103. 3 Un kinésithérapeute devra effectuer l'évaluation initiale, le plan de soins et les réévaluations,
les modifications du plan de soins et les plans de sortie pour le patient. Le patient qui est stable et qui a un
programme existant serait le patient le plus approprié pour être délégué à l'assistant. Par conséquent, le patient
atteint de sclérose en plaques et ayant suivi un programme d'entraînement à la marche serait le plus approprié. Les
trois autres patients reçoivent leur traitement initial.

104. 2 L'option 2 est la définition de la validité. Les options 3 et 4 font référence à des mesures de fiabilité.
L'option 1 est incorrecte
parce que les résultats peuvent être standardisés même s'ils ne sont pas valides.

105. 4 Une rampe continue de 60 pieds (18 m) de long est trop longue sans zone de repos plane. Un
Une rampe continue de 30 pieds (9 m) de long présente une pente trop raide. Deux rampes de 18 m de long
chacune, même si elles sont reliées par une zone plane, seraient inutilement longues. L'élévation ne doit pas être
supérieure à 76 cm (30 in) pour chaque passage et doit être de 0,3 m (1 ft) de longueur pour chaque 2,5 cm (1 in)
d'élévation.

106. 4 Les exercices de résistance sont contre-indiqués chez les patients ayant des antécédents d'anévrisme
aortique, même après une intervention chirurgicale.
réparation.

107. 2 L'épicondylite latérale est causée par une surutilisation des extenseurs du poignet qui prennent naissance
sur l'épicondyle latéral de la tête.
l'humérus, en particulier l'extensor carpi radialis brevis. Si l'épicondylite latérale est à un stade chronique, le
conditionnement des muscles extenseurs et les activités de préhension soutenues seront les plus efficaces pour une
prise en charge à long terme. On pense qu'une manchette d'avant-bras diminue la charge musculaire. L'ionophorèse
n'est pas appropriée pour un programme à domicile. Le massage par friction du muscle brachio-radial n'est pas
approprié, car c'est généralement le muscle extenseur radial qui est affecté.

108. 1 Lorsque les patients présentent des symptômes sévères, il est souvent utile d'augmenter les périodes
d'activation et de désactivation afin de réduire le risque d'infection.
le mouvement causé par le cyclisme. L'augmentation du temps d'arrêt permet de prolonger le temps de récupération
avant de relancer la traction et de réduire le cyclage. Au fur et à mesure que les symptômes de la douleur
diminuent, le temps de relaxation peut être réduit. Les options 2, 3 et 4 ne prévoient pas de période de repos ou de
récupération plus longue.

PEAT
4
Réponses 39

109. 1 La plage normale des gaz du sang artériel est un pH compris entre 7,35 et 7,45. Un pH de 7,35 ou moins
indique
l'acidémie, également appelée état d'acidose.

110. 4 Le signe initial le plus courant est une hypertrophie du testicule accompagnée d'une douleur testiculaire
diffuse, d'un gonflement ou de l'apparition d'une tumeur.
dureté. Des douleurs dorsales peuvent être présentes, ainsi que des douleurs référées à l'aine ou un gonflement des
ganglions lymphatiques superficiels, mais la douleur et le gonflement des testicules sont les symptômes
prédominants les plus fréquents au début.

111. 3 L'option 1 ne mentionne pas la position du genou et la hanche est fléchie, ce qui raccourcit le rectus
femoris.
Dans l'option 2, la hanche est fléchie, ce qui raccourcit le muscle droit du fémur. La position couchée avec le genou
en flexion maintient la hanche en position neutre et ne permet pas à la hanche de se fléchir. Cette position est
utilisée dans le test Ely. L'option 4 présente la hanche dans la position correcte, mais ne mentionne pas la position
du genou.

112. 4 La position fermée de l'articulation radiocarpienne est l'extension complète avec déviation radiale. La
fermeture
La position d'emballage de l'articulation carpienne moyenne est l'extension avec déviation radiale.

113. 2 Les béquilles axillaires, le déambulateur standard et les béquilles d'avant-bras nécessitent tous une force
suffisante au niveau du coude
les extenseurs et le latissimus dorsi, notamment en raison du fait que le patient ne porte pas de poids. L'absence de
port de poids à droite () pourrait indiquer l'une ou l'autre des options ; cependant, comme le patient présente une
faiblesse des muscles dudermatome C6-C7, il aurait des difficultés à utiliser tout appareil d'assistance nécessitant
l'utilisation des extenseurs du coude et des muscles du latissimus dorsi. La meilleure option est donc le fauteuil
roulant, car il permet de se déplacer.

114. 2 Les seuils de pression utilisant des filaments de nylon sont les plus sensibles et les plus spécifiques.
Plusieurs études soutiennent la
l'utilisation du filament de nylon de 10 grammes (Semmes-Weinstein 5,07) comme seuil de la sensation de
protection. Les patients incapables de sentir un filament de nylon de 10 grammes sont considérés comme
incapables de protéger leurs pieds contre les blessures et sont exposés à un risque d'ulcération.

115. 2 Une vue d'ensemble présentée le premier jour impliquerait que le thérapeute ne va pas modifier la série
pour
le niveau de connaissance actuel des étudiants, qui peut être déterminé à partir du pré-test. Il est très important pour
le kinésithérapeute de connaître le niveau de connaissances des étudiants. Bien que la participation active soit
importante pour l'apprentissage dans chaque session, ni elle, ni une démonstration complète d'une évaluation du bas
du dos, ne sont les plus importantes à inclure dans la première session.

116. 1 Un score standard, ou score z, se rapporte au nombre d'unités d'écart-type qu'un score se situe au-dessus ou
au-dessous d'une norme.
moyenne. Les scores standard sont souvent utilisés dans les mesures de développement parce qu'ils permettent une
comparaison utile avec un groupe plus important ou normal. Un score standard de -2,0 est nettement inférieur à la
moyenne, car deux unités d'écart-type en dessous de la moyenne indiquent que seuls 2,27% des enfants ont obtenu
un score inférieur ou, inversement, que 97,73% des enfants ont obtenu un meilleur score. À -1,0 unité d'écart-type,
84,14 % des enfants ont obtenu de meilleurs résultats.

117. 4 Le genou médial est le dermatome L3. Le creux poplité est le dermatomeS2. Le pied plantaire estS1 ouS2. A
un renflement postéro-latéral du disque L4-L5affectera très probablement la racinenerveuse L5 et le dermatome de la
racinenerveuse L5. Ledermatome L5 comprend la face dorsale du gros orteil. La zone dorsale du gros orteil est
systématiquement incluse dans ledermatome L5.

118. 1 Par définition, la sténose lombaire provoque des symptômes au niveau des membres inférieurs lors des
activités d'extension.
plus fréquente dans une population plus âgée. La claudication de l'artère glutéale provoque une douleur dans la
fesse et non dans le mollet. Ce patient est relativement trop âgé pour souffrir d'une hernie du noyau pulposus, et la

PEAT
4
40 Réponses

plupart des cas de hernie du noyau pulposus n'entraînent pas de symptômes bilatéraux. Rien dans les antécédents
du patient n'augmente la probabilité d'une thrombose veineuse profonde.

119. 3 Le mot clé de cette question est "interdisciplinaire". De toutes les options proposées, seule la libération des
voies aériennes
fait partie du champ d'activité des kinésithérapeutes. Les patients atteints du syndrome de Guillain-Barré peuvent
ressentir une fatigue ou une paralysie des muscles respiratoires et sont sensibles aux infections pulmonaires.
L'hygiène pulmonaire joue un rôle essentiel dans leur prise en charge.

120. 1 Le thérapeute effectue un glissement ulnaire, qui est le même mouvement articulaire que celui utilisé pour
le glissement radial.
déviation. Un mouvement limité dans cette direction indique une capacité limitée à effectuer une déviation radiale.

121. 2 Bien que toutes les options constituent des complications graves chez les patients atteints du sida, la
maladie multifocale est la plus grave.
La leucoencéphalopathie est la plus grave et la mort survient quelques mois après le diagnostic.

122. 1 Une mauvaise turgescence de la peau est l'un des signes de déshydratation. La peau, lorsqu'elle est
soulevée entre les doigts, n'est pas
revenir rapidement à sa position initiale.

123. 4 Un objectif bien rédigé comprend un public (qui), un comportement (ce qui sera fait), une condition (sous
quelle forme).
) et le degré (importance du changement). La seule option qui inclut toutes ces conditions est l'option 4, "Le patient
(qui) démontrera une force musculaire thénar (comportement) de bon (4/5) niveau (degré) et sera capable de tenir
des objets dans la main au travail (condition)". Dans les options 1, 2 et 3, il manque un ou plusieurs des éléments
requis.

124. 4 Le transfert debout-pivot nécessite de se tenir debout, ce que le patient n'est pas en mesure de faire en toute
sécurité. Pivot manuel
le transfert implique l'assistance d'une autre personne et ne permet donc pas l'indépendance. Le transfert par lève-
personne mécanique implique l'assistance d'une autre personne et ne permet donc pas l'indépendance. Le transfert
par glissement est la technique indépendante la plus sûre, car le patient ne peut pas se tenir debout sans une aide
minimale.

125. 3 Le soulagement de la douleur en se penchant en avant ou en s'asseyant et le soulagement de la douleur en


se levant sont des signes plus diagnostiques d'une douleur lombaire.
l'origine de la douleur. Une douleur de type crampes survenant à une distance de marche prévisible est une
présentation courante et une manifestation clinique de la limitation vasculaire. Les engourdissements et les
picotements qui se produisent à une distance prévisible de la marche indiquent davantage une cause neurologique
de la douleur qu'une cause vasculaire.

126. 2 Étant donné que le patient ne présente pas de symptômes lorsqu'il regarde droit devant lui, l'acuité visuelle
(acuité périphérique) a été calculée à l'aide d'une méthode de mesure.
système visuel) ne semble pas être en cause. Un patient qui se plaint d'un flou visuel lorsqu'il bouge la tête souffre
d'oscillopsie. L'oscillopsie est une instabilité visuelle accompagnée d'un mouvement de la tête dans laquelle les
images semblent bouger ou rebondir. Elle est souvent due à une diminution du réflexe vestibulo-oculaire. Aucune
information n'est donnée dans la question pour suggérer que le patient a des difficultés avec l'entrée
somatosensorielle ou qui indique des déficits musculo-squelettiques.

127. 4 L'option 1 est un dispositif d'assistance inapproprié pour un jeune adulte paraplégique. L'option 2 est
l'ultime
L'objectif de l'entraînement à la marche est d'améliorer la qualité de vie du patient, mais il serait trop difficile pour
la première tentative du patient. L'option 3 n'est pas l'aide à la marche la plus efficace pour un patient paraplégique.
La première séance d'entraînement à la marche pour un patient paraplégique doit se faire aux barres parallèles. Un
schéma de marche de type "swing-to" serait le plus facile à apprendre pour le patient au début.

128. 4 Des étirements doivent être effectués pendant et immédiatement après le traitement par ultrasons. 1 MHz
continu

PEAT
4
Réponses 41

Les ultrasons produisent les effets thermiques les plus importants et facilitent l'étirement.

129. 2 La dyspnée paroxystique nocturne est fréquente en cas de maladie cardiaque grave. Soulager les
symptômes de l'essoufflement
par un changement de position suggère que le problème est d'origine pulmonaire. Un essoufflement occasionnel
peut être attendu lors d'un changement d'activité physique, mais il convient de recueillir davantage d'informations
sur les antécédents afin de déterminer si le changement est attendu. Les vertiges sont plutôt le signe d'un problème
cardiaque.

130. 1 La compréhension du niveau de fonctionnement antérieur du patient est le facteur le plus important dans
les antécédents du patient.
déterminer un pronostic raisonnable, car ces informations établissent la base de la guérison. Bien que l'amplitude
des mouvements du genou et l'intégrité sensorielle des membres supérieurs doivent être contrôlées, aucune de ces
deux données n'est plus importante que le niveau de fonctionnement antérieur en tant que facteur déterminant du
pronostic. Bien que le collier cervical soit une intervention potentielle, son utilisation n'est pas plus déterminante
que le niveau de fonction antérieur pour le pronostic.

131. 1 L'articulation de Charcot est indolore et n'est généralement pas détectée par le patient en raison d'une
diminution de la sensation et de l'intensité de la douleur.
neuropathie. En raison de l'augmentation du flux sanguin, les patients présentent un érythème.

132. 2 Une évaluation complète de la kinésithérapie et un traitement sont contre-indiqués en raison d'un risque de
maladie veineuse profonde.
thrombose. Un examen Doppler prescrit par le médecin indique la possibilité d'une thrombose veineuse profonde.
La kinésithérapie ne doit donc pas être effectuée tant que l'examen Dopler n'a pas été réalisé et que les résultats
n'ont pas été analysés par le médecin. Le transfert du lit à la chaise est contre-indiqué en raison du risque de
thrombose veineuse profonde. L'infirmière ne doit pas être la seule à donner l'autorisation, tant que l'étude Doppler
n'a pas été réalisée et interprétée.

133. 2 L'objectif de l'option 2 est le seul qui soit objectif, mesurable et fonctionnel. Les objectifs des options 1,
3, et 4 manquent de fonction ou d'informations objectives spécifiques et mesurables.

134. 3 Lorsque le côté droit du cœur commence à faiblir, du liquide s'accumule dans les pieds et la partie
inférieure des jambes, ce qui provoque des piqûres.
œdème. La pâleur, la peau brillante et l'absence de croissance des cheveux sont caractéristiques de la maladie
artérielle, mais ne sont pas associées à l'insuffisance cardiaque congestive.

135. 4 La présence de tissu nécrotique noir indique une perte totale de l'épaisseur de la peau. La cause de la
nécrose
ne peut pas être déterminée avec les informations contenues dans la tige.

136. 4 Si une personne commence à faire de l'exercice alors que sa glycémie est inférieure à 100 mg/dL ou
supérieure à 250 mg/dL, il est possible qu'elle subisse des dommages importants.
des complications liées à l'exercice peuvent en résulter.

137. 2 Les changements neurologiques liés à l'âge comprennent une diminution de la douleur et des sensations
vibratoires. L'autre
changements répertoriés qui ne se produisent pas normalement avec le vieillissement en l'absence de pathologie.

138. 1 L'apparition d'une escarre de stade I se caractérise par un érythème non décoloré et une diminution de
l'épaisseur de la peau.
température sur le site de l'ulcère. Le blanchiment étant difficile à déterminer chez les patients à la peau foncée, il
est préférable d'utiliser la température de la peau pour évaluer les escarres.

139. 2 Dans une expérience à facteur unique, l'analyse de variance à une voie est appliquée lorsque trois ou plus
de trois facteurs sont présents.
les moyennes des groupes indépendants sont comparées. Le descripteur unidirectionnel indique que le modèle
implique une seule variable indépendante, ce qui est le cas dans le présent scénario. Tous les sujets ont été recrutés
dans la même clinique et ont subi la même durée de traitement. La seule différence entre les trois groupes de sujets

PEAT
4
42 Réponses

était le groupe de traitement auquel ils étaient assignés.

140. 3 Le nerf oculomoteur (III) contrôle les muscles oculaires inféromédiaux. Le nerf trochléaire (IV) contrôle
mouvement oculaire inféro-latéral. Le nerf abducens (VI) contrôle les mouvements latéraux de l'œil. Les lésions de
ce nerf entraînent une déviation médiale du globe oculaire en raison de la faiblesse du muscle droit latéral. Le nerf
vague (X) ne contrôle pas les muscles oculaires.

141. 2 Comme le patient présente une faiblesse importante au niveau des deux membres inférieurs, un transfert en
position debout et en pivot est nécessaire.
ne sont pas sûres. Cependant, étant donné que le patient a une force moyenne ou normale au niveau des membres
supérieurs et qu'il n'est pas totalement dépendant, un lève-personne hydraulique ou un lève-personne pour deux
personnes ne serait pas approprié car il n'améliorerait pas les capacités fonctionnelles du patient. La méthode de
transfert la plus appropriée serait que le patient utilise une planche coulissante sur le côté le plus fort (droit).

142. 3 L'option 3 est la seule à être clairement énoncée en termes éducatifs. Les autres options sont des objectifs
basés sur
observations.

143. 1 L'atélectasie se produit lorsqu'un ou plusieurs segments ou lobes du poumon sont affaissés. Elle survient
souvent à la suite
chirurgie thoracique ou abdominale supérieure, lorsque le patient présente une capacité pulmonaire totale, une
capacité résiduelle fonctionnelle et un volume résiduel réduits. La bronchectasie est une dilatation anormale des
bronches et des bronchioles qui entraîne une augmentation des volumes résiduels. Le principal symptôme de la
bronchite chronique est une toux productive chronique qui se traduit par une augmentation du volume résiduel.
L'emphysème se traduit généralement par une augmentation significative des volumes résiduels.

144. 3 Pendant la grossesse et le post-partum, les muscles abdominaux étirés sont incapables de stabiliser le bas
du dos.
au fur et à mesure que les jambes sont soulevées. En essayant d'effectuer des levées de jambes doubles, vous
risquez de surmener les muscles abdominaux et d'endommager les articulations de la colonne vertébrale.

145. 1 L'apprentissage précoce au cours de la première phase de l'apprentissage moteur (la phase cognitive)
nécessite une attention totale à la tâche.
et le feedback extrinsèque. Au fur et à mesure de l'apprentissage de la tâche, le retour d'information peut devenir
plus intrinsèque et l'attention portée à la tâche peut diminuer.

146. 1 L'option 1 teste l'intégrité de la syndesmose tibio-fibulaire. En cas de résultat positif, le patient
ont reproduit les symptômes et, dans le cas présent, la douleur antérieure de la cheville. La rotation latérale
(externe) du tibia peut potentiellement comprimer l'articulation calcanéocuboïdienne mais ne sollicite pas les
ligaments. Une fracture du5e métatarsien ne présente pas de douleur antérieure à la cheville. L'option 4 ne sollicite
pas suffisamment le tibia et/ou les muscles pour provoquer une douleur due à une fracture de stress.

147. 4 Pendant la mesure de l'abduction de l'épaule, le tronc doit rester droit. Si le patient est autorisé à
une flexion latérale du tronc donnera l'impression d'une augmentation de l'abduction de l'épaule, mais le
mouvement se produira dans la colonne vertébrale et non dans l'épaule. La rotation vers le haut de l'omoplate et la
rotation latérale (externe) de l'épaule accompagnent normalement le mouvement d'abduction de l'épaule. La
rotation médiale (interne) de l'épaule diminue l'abduction disponible.

148. 3 La capacité vitale est mesurée de l'inspiration maximale à l'expiration maximale. Capacité inspiratoire
mesure le volume lors d'une inhalation maximale. Le volume courant mesure le volume total d'air déplacé pendant
l'inspiration ou l'expiration sur une période de temps spécifique (généralement 1 minute), puis divisé par la
fréquence ventilatoire. Le volume de réserve inspiratoire mesure le volume inspiratoire au-delà de l'inspiration
normale.

149. 3 Le réservoir doit être placé sous le niveau du cœur. Il suffit d'en informer l'infirmière.
Il n'est pas nécessaire d'informer le médecin.

150. 3 L'extension du genou en décubitus dorsal avec flexion de la hanche étire le nerf sciatique, qui implique les
muscles lombaires inférieurs.

PEAT
4
Réponses 43

racines nerveuses (L4-S3). La flexion du genou avec la flexion de la hanche n'étire pas le nerf fémoral. Une
radiculite lombaire supérieure touche les racines nerveuses qui forment le nerf fémoral (L2-L4). Le nerf fémoral
peut être étiré ou provoqué en étendant la hanche tout en fléchissant le genou. L'extension de la hanche en
décubitus ventral étire le nerf fémoral, mais l'étirement est moins important lorsque le genou est en extension qu'en
flexion.

151. 4 Une position assise normale nécessite une flexion de la hanche d'environ 105°, de sorte qu'une chaise
surélevée ne serait pas nécessaire. Atteindre
ne serait pas aussi limité que le port de chaussettes. Le reacher ne serait pas efficace pour aider à porter des
chaussures et des chaussettes et pourrait être surutilisé. L'ascension d'un escalier nécessite une flexion de la hanche
de 60 à 70°, et il n'est pas nécessaire de modifier la hauteur des marches. Pour atteindre le pied, il faut une flexion
de la hanche d'environ 120°. Si une procédure modifiée est choisie avec le pied en travers de la cuisse opposée, une
flexion de la hanche d'au moins 110° est toujours nécessaire. Avec seulement 105° de flexion de la hanche, le
patient aurait le plus grand mal à enfiler des chaussettes.

152. 3 Pendant la phase d'élan de la marche, le pied est dorsiflexé pour se dégager du sol. Muscles actifs pendant
cette phase
de la marche sont le tibialis anterior, l'extensor hallucis longus et l'extensor digitorum longus. Le gastrocnémien, le
tibialis posterior et le fibulaire (peroneus) longus sont actifs pendant la phase d'appui de la marche. Par conséquent,
parmi les muscles cités, l'extensor digitorum longus est le meilleur choix. Si seul le tibialis anterior était stimulé, le
pied entrerait en inversion et en dorsiflexion, ce qui pourrait amener le patient à atterrir sur le côté latéral du pied
lors du contact initial, provoquant ainsi une blessure. L'ajout de l'extensor digitorum longus amènerait le pied à une
plus grande dorsiflexion et éviterait une inversion excessive.

153. 3 Le modèle le plus approprié serait un prétest-post-test à deux groupes. Les deux groupes sont mesurés à la
même temps, mais seul le groupe expérimental reçoit le traitement. La collecte d'informations à partir des dossiers
des patients peut ne pas fournir la même précision que si le chercheur avait effectué toutes les mesures directement.
Le fait de mesurer la ROM uniquement le cinquième jour n'expliquerait pas les différences de pré-test entre les
groupes. Le fait de mesurer le groupe expérimental plus fréquemment que le groupe de contrôle peut avoir un effet
sur la variable dépendante.

154. 4 Les patients doivent être encouragés à exprimer leurs sentiments. Comparaisons avec d'autres patients en
fin de vie, en
un effort pour assurer le patient qu'il n'est pas seul, ne tient pas compte des sentiments de ce patient. Le déni de la
mort ne serait pas bon pour le patient, puisqu'il doit finalement faire face à l'inévitable. Le conseil pastoral peut être
une option, mais le thérapeute doit être prêt à écouter le patient, à encourager l'expression de ses sentiments et à
éviter le déni.

155. 4 La localisation de l'ulcère indolore est caractéristique d'un ulcère neuropathique (diabétique). Perte de
sensibilité
rend le patient inconscient des traumatismes répétitifs dus à l'altération de la biomécanique en raison de la faiblesse
des muscles intrinsèques, ce qui entraîne des orteils en marteau et une pression anormale sur le pied plantaire lors
de la mise en charge. L'augmentation de la douleur lors de l'élévation du pied est caractéristique des ulcères
artériels, qui sont douloureux. Des pouls artériels normaux peuvent être présents en cas d'ulcère veineux, mais les
artères sont souvent calcifiées en cas de diabète avancé. Un pied neuropathique est chaud et sec.

156. 1 Le thérapeute effectue un glissement palmaire, qui est le même mouvement articulaire que celui utilisé
pour la flexion des doigts.
Un mouvement limité dans cette direction indique une capacité limitée à effectuer une flexion des doigts.

157. 3 L'un des avantages de l'exercice physique est l'amélioration de la sensibilité à l'insuline, qui réduit la
quantité d'insuline.
nécessaires. La glycémie diminue pendant l'exercice. Les patients diabétiques ne doivent pas faire d'exercice avant
les repas, car ce moment augmente le risque d'hypoglycémie. L'exercice aérobique est tout aussi important pour les
personnes atteintes de diabète que pour les personnes en bonne santé, et il n'est donc pas préférable de faire de la
musculation.

158. 3 Tous ces symptômes sont révélateurs du syndrome douloureux régional complexe. La discopathie
cervicale n'est pas

PEAT
4
44 Réponses

produisent un gonflement de la main, des changements de couleur et de température, ou une hyperhidrose. Le


phénomène de Raynaud se traduit par une douleur, une pâleur et une sensation de fraîcheur, mais pas
d'hyperhidrose. Bien que le syndrome du canal carpien puisse également présenter des anomalies du système
nerveux sympathique, la compression du nerf médian entraînerait des symptômes au niveau du bord latéral de la
main (région du pouce). En cas de syndrome du canal carpien, le patient peut également présenter une faiblesse du
muscle thénar.

159. 1 L'évaluation de la rétention par les patients des informations présentées dans le programme peut être
améliorée par les moyens suivants
poser des questions aux patients sur les informations relatives au programme, demander aux patients de poser des
questions sur le programme, demander aux patients de montrer ce qu'ils ont appris et tester les patients sur le
matériel du programme. L'option 1 est la seule à inclure l'une de ces techniques. Les autres options ne requièrent
pas la participation active des patients et ne prévoient pas d'interventions spécifiques.

160. 1 Les troubles inflammatoires chroniques tels que la polyarthrite rhumatoïde sont souvent associés à des
douleurs matinales et à un manque de sommeil.
raideur qui diminue au cours de la journée avec l'activité. Une douleur qui s'aggrave au début d'une activité
implique une inflammation aiguë. Une douleur constante suggère des tumeurs ou une cause viscérale. Une douleur
qui augmente tout au long de la journée indique une congestion accrue dans une articulation.

161. 1 Le massage à la glace est utilisé pour traiter une petite zone, par exemple un point de déclenchement dans
un muscle. La glace est appliquée pour produire
L'utilisation de ces produits permet d'obtenir un effet analgésique et d'exercer une pression profonde sur le point
gâchette et d'étirer le muscle. Le traitement de grandes surfaces, comme la partie inférieure de la jambe (comme
dans l'option 2), est plus efficace avec des bains froids ou des packs de froid qu'avec un massage à la glace. La
cryothérapie augmente la rigidité et diminue l'extensibilité des tissus (p 76) et ne serait donc pas indiquée pour
étirer les ischio-jambiers.

162. 2 La névralgie du trijumeau provoque des douleurs faciales. Tous les signes et symptômes mentionnés dans
la question sont révélateurs d'une
lésion nerveuse partielle affectant le nerf facial. Ce type de lésion est la paralysie de Bell. Un accident vasculaire
cérébral gauche n'affecte que les muscles faciaux inférieurs sous l'œil. Les lésions du nerf oculomoteur entraînent
la paralysie d'un ou de plusieurs des muscles qui font bouger le globe oculaire.

163. 3Toutes les options, à l'exception des exercices doux et actifs, sont des précautions ou des contre-indications pour
ce patient.

164. 4La validité externe concerne la question de la généralisation des résultats de la recherche.

165. 1À ce stade de l'hémarthrose, il y a encore un saignement dans l'interligne articulaire, mais il n'est pas aussi
important que dans le cas de l'hémarthrose.
pendant la phase aiguë. Par conséquent, le patient bénéficiera d'exercices d'amplitude de mouvement pour prévenir
la contracture. Le patient peut avoir besoin d'une assistance active, car il peut encore y avoir une douleur ou un
œdème dans l'articulation qui empêche l'exécution indépendante de l'amplitude des mouvements. À ce stade, le
traumatisme mécanique de l'appui du poids sur la tolérance peut toucher et endommager la synovie pathologique à
l'intérieur de l'articulation. L'amplitude de mouvement résistive est plus appropriée lorsque la douleur et le
gonflement ont disparu et qu'il n'y a pas de saignement. L'immobilisation continue en position étendue favorise la
contracture du genou œdémateux.

166. 1 L'échelle de Glasgow est utilisée comme mesure générale des résultats et dans les études pronostiques. Le
Fugl-
L'évaluation Meyer ne doit être utilisée que pour les patients ayant subi un accident vasculaire cérébral. L'échelle
des niveaux de fonctionnement cognitif de Rancho Los Amigos est une échelle descriptive qui décrit une séquence
prévisible de récupération cognitive et comportementale lorsqu'un patient sort du coma. Le profil d'impact de la
maladie est utilisé pour mesurer l'état de santé général et est auto-administré ; par conséquent, il ne serait pas utile
comme outil de pronostic de sortie pour un patient souffrant d'une lésion cérébrale traumatique.

167. 3 La cauda equina provoque un dysfonctionnement des motoneurones inférieurs, y compris la rétention
urinaire et l'incontinence.

PEAT
4
Réponses 45

Des réponses positives de Babinski et de clonus indiquent un dysfonctionnement du motoneurone supérieur. Un


test de flexion du genou en décubitus ventral positif est plus probable en cas d'atteinte des racines nerveuses
lombaires supérieures.

168. 1 Dans le cas d'un programme d'exercices d'extension, la centralisation indique que le patient s'améliore,
même si
la douleur peut être aussi intense qu'elle l'était. Même si le patient se sent frustré, il s'améliore et le thérapeute doit
donc poursuivre le programme d'exercices d'extension. Il n'est pas nécessaire d'appeler le médecin à ce stade, car la
centralisation est parfaitement normale. Bien que la flexion du tronc puisse être utilisée plus tard dans
l'intervention, elle n'est pas indiquée à ce stade car l'extension fonctionne.

169. 4 Les étirements contracter-relaxer pour les structures tendues ne sont pas recommandés pour les tissus
mous fragilisés par
la polyarthrite rhumatoïde. Les techniques de mobilisation de grade IV sont contre-indiquées pour les tissus mous
fragilisés par la polyarthrite rhumatoïde. L'élimination des activités fonctionnelles n'est pas nécessaire. Les
exercices de renforcement sont l'intervention la plus appropriée pour un patient atteint de polyarthrite rhumatoïde
en rémission.

170. 3 Une douleur accrue à la palpation de la paroi thoracique est plus révélatrice d'une origine musculo-
squelettique de la douleur. Augmenté
une douleur lors d'une respiration profonde est plus révélatrice d'une origine pulmonaire de la douleur. La
nitroglycérine (Nitrostat) est un vasodilatateur courant qui est prescrit aux patients souffrant d'angine de poitrine.
L'ingestion d'un vasodilatateur améliorera le flux sanguin myocardique et contribuera à soulager l'ischémie et ses
manifestations. Le soulagement de la douleur par l'ingestion d'antiacides est plus révélateur d'une douleur référée
provenant d'un ulcère gastroduodénal.

171. 1 L'antéversion fémorale excessive est la cause la plus fréquente de l'orientation vers l'intérieur ou
"strabisme" de la rotule.
Bien que l'antéversion fémorale soit normalement de 8° à 15°, une antéversion excessive entraîne un plissement de
la rotule et un pincement du pied. Les autres options auraient toutes tendance à faire pointer les rotules vers
l'extérieur en position debout.

172. 4 Une mesure de résultat documente une performance souhaitée ou un changement dans l'état du patient au
fil du temps.
Il peut s'agir d'une description de la fonction du patient avant, pendant et après l'intervention. L'option 4 n'indique
pas de changement ou de fonction, mais se concentre sur l'assiduité.

173. 3 La section complète de la moelle épinière entraîne une perte motrice et sensorielle des deux côtés du corps.
Une lésion
des cellules de la corne antérieure provoque un problème de motoneurones inférieurs et une hyporéflexie. Les
signes et symptômes présentés dans la question indiquent une hémisection de la moelle épinière du côté gauche, ce
qui entraîne une perte de motricité et de proprioception ipsilatérale et une perte de douleur et de température
controlatérale. On note également une hyperréflexie et un signe de Babinski positif du côté gauche. Une lésion de
la moelle épinière centrale épargne généralement les voies motrices.

174. 1 L'irradiation de la cavité pelvienne provoque souvent des adhérences pelviennes denses qui peuvent limiter
les mouvements en raison de la douleur. De
Parmi les options, les exercices d'étirement sont les plus spécifiques pour traiter cette complication.

175. 4 Au fur et à mesure que la fréquence des visites diminue et que l'intervalle entre les visites augmente, le
patient a besoin d'une plus grande quantité d'oxygène.
un programme détaillé à domicile avec des instructions de progression spécifiques.

176. 2 En ce qui concerne l'option 1, un ratio de 75 % est considéré comme normal. En ce qui concerne l'option 2,
le patient a une légère
maladie. Une maladie obstructive avancée peut atteindre des valeurs de 25 %. Néanmoins, le patient présente une
légère morbidité et il convient donc de prendre des précautions particulières. En ce qui concerne l'option 3, le
patient souffre d'une maladie bénigne et ne présente pas encore de signes de morbidité, de sorte que des précautions
particulières ne devraient pas être nécessaires. Il n'est pas nécessaire de limiter la fréquence cardiaque à 60 % si le

PEAT
4
46 Réponses

patient tolère l'exercice. Un rapport VEMS/capacité vitale forcée de 60 % est une perte légère. Il n'est pas nécessaire
d'éviter les exercices aérobiques, ni d'obtenir l'approbation spéciale d'un médecin pour une valeur de 65 %.

177. 2La jaunisse est un signe de maladie du foie.

178. 1Les patients atteints du sida sont ceux qui ont le VIH-1et un taux de CD4 inférieur à 200 cellules/mL. Un faible
La numération des neutrophiles ou un titre ANA ou HLA élevé ne sont pas spécifiques de la présence du VIH.

179. 3La thrombocytopénie est une diminution aiguë ou chronique de l'activité de l'organisme.du nombre de plaquettes
dans la circulation. LeT4
La numération des lymphocytes est utilisée pour évaluer l'état immunitaire des patients atteints du VIH ou du sida.
La numération des globules rouges est utilisée pour évaluer la présence d'une anémie, et la numération des globules
blancs est examinée pour déterminer la présence d'une infection ou le degré d'immunosuppression.

180. 3 L'hypertension artérielle, le tabagisme et l'hyperlipidémie sont des facteurs de risque directs ou primaires
de la maladie.
l'athérosclérose. Les facteurs de risque secondaires sont l'âge, le sexe, la race, l'obésité, le stress et le niveau
d'activité.

181. 2 L'option 1 ne précise pas le montant de l'aide. L'option 2 est un objectif mesurable, spécifique et objectif.
L'option 3 ne précise pas le dispositif que le patient utilisera. L'option 4 est un objectif à atteindre d'ici à la
décharge, ce qui serait un objectif à long terme et non à court terme.

182. 2 Les muscles du plancher pelvien sont innervés par les racines nerveuses sacrées S2-S4via le nerf pudendal. Les
Le nerf fessier inférieur est un nerf moteur qui innerve le grand fessier. Le nerf sciatique contient des racines
nerveuses allant de L4à S3et innerve plusieurs muscles de la partie inférieure de la jambe. Le nerf génito-fémoral,
qui se situe entreL1 etL2, est un nerf cutané sensitif et non moteur.

183. 2 Les malaises et la fatigue sont des symptômes courants au début de la polyarthrite rhumatoïde. Tension
les maux de tête, l'intolérance à la chaleur, les nausées et les diarrhées ne sont pas associés à la polyarthrite
rhumatoïde.

184. 3 Il a été démontré que l'exercice physique augmente la sensibilité des récepteurs de l'insuline, ce qui
entraîne une diminution du taux d'insuline.
la quantité d'insuline nécessaire. L'administration de médicaments est dictée par la tolérance et l'efficacité du
médicament et ne serait pas nécessairement modifiée par l'exercice. Chez les patients présentant une hyperglycémie
modérée, l'exercice peut entraîner une hypoglycémie pendant les 24 à 48 heures suivant l'exercice ; par conséquent,
une augmentation de l'apport calorique, en particulier de l'apport en hydrates de carbone, serait essentielle.

185. 2 Le patient a trop d'éversion pendant la phase intermédiaire de l'élan. Cela est probablement dû à un excès
de
l'activité de l'extensor digitorum longus ou de l'extensor hallucis longus. Pour corriger la situation, il faut faciliter le
tibialis anterior, car il est dorsiflexe et inversé, mais ne provoque pas d'éversion. Le fibularis brevis, un éverseur,
provoquerait une flexion plantaire du pied plutôt qu'une dorsiflexion. Le flexor hallucis longus ne dorsifie pas la
cheville.

186. 3 Les plaies se cicatrisent par un processus complexe de ré-épithélialisation. Avec l'avancée en âge, le taux
de
la prolifération épidermique diminue.

187. 3 L'option 1 ne pose pas de question sur la fonction. Il s'agit d'une question liée à la déficience et non à la
fonction.
question. Les options 2 et 4 ne posent pas de questions sur la fonction et, même en cas d'amélioration, aucune
n'aurait de rapport avec la fonction. L'option 3 est tirée de l'évaluation succincte de la fonction musculo-
squelettique. Elle porte sur la fonction et peut être utilisée comme question de résultat.

188. 2 Pour ce patient, la première procédure de mobilisation serait la distraction de l'articulation gléno-humérale.
Les
La distraction sépare les surfaces articulaires et est utilisée pour tester le jeu des articulations. La distraction peut

PEAT
4
Réponses 47

également contribuer à augmenter le jeu en commun. La distraction peut également être utilisée en conjonction
avec les autres techniques de mobilisation énumérées. Les techniques de mobilisation ultérieures incluront
probablement un glissement antérieur.

189. 2 Parmi les interventions énumérées, la seule qui nécessite des précautions en raison de la pose d'une
prothèse totale de la hanche est la suivante
ultrasons en continu. Toutefois, cela ne signifie pas que l'échographie est contre-indiquée pour ce patient. La
stimulation nerveuse électrique transcutanée peut être utilisée sur des implants métalliques. Les compresses
chaudes et les massages n'affectent pas la prothèse totale de hanche.

190. 1 Afin d'éliminer au mieux la congestion qui s'est développée dans le lobe inférieur droit, le patient doit être
positionné sur le côté gauche pour permettre à la gravité de drainer les sécrétions vers les voies respiratoires
proximales. Ce patient est également inconscient et intubé, donc en plus du drainage, une hyperinflation manuelle
est nécessaire pour augmenter la ventilation afin d'aider à mobiliser les sécrétions, puis une aspiration pour éliminer
les sécrétions. L'aspiration d'abord, puis l'hyperventilation en position latérale gauche sont moins efficaces que
l'ordre suggéré dans l'option 1. Le positionnement sur le côté droit ne permettrait pas de drainer le lobe inférieur
droit, et comme le patient est allongé sur le côté droit, le seul endroit où percuter et vibrer serait le côté gauche, qui
ne présente pas de pathologie. Enfin, le positionnement sur le côté gauche avec des exercices de respiration
profonde ne sera pas efficace pour mobiliser les sécrétions, d'autant plus que le patient est inconscient et incapable
d'effectuer activement des exercices de respiration profonde.

191. 3 Entraînement et retour d'information sur les compétences d'examen, jeu de rôle sur le processus d'entretien,
et formation à son propre rythme.
impliquent un apprentissage actif avec d'autres personnes dans des situations de la vie quotidienne. L'apprentissage
actif repose sur le sentiment et l'intuition plutôt que sur la logique et le raisonnement. Les lectures ne répondent pas
aux besoins d'apprentissage de l'expérimentation active ou de l'expérience concrète.

192. 4 En raison des adaptations circulatoires nécessaires, les patients souffrant d'hypertension non contrôlée ne
seraient pas
autorisés à utiliser des techniques d'immersion totale. L'hydrothérapie en immersion totale permet d'améliorer
l'œdème dû aux effets de la pression de l'eau, de réduire la gravité de l'asthme induit par l'exercice par rapport aux
activités terrestres et de réduire le stress sur les zones squelettiques en cas d'ostéoporose.

193. 1 L'hypothyroïdie est très probablement à l'origine des signes et des symptômes. L'hyperthyroïdie provoque
des diarrhées,
plutôt que la constipation. L'hyperparathyroïdie n'est pas associée à la bradycardie. L'hypoparathyroïdie est
associée à des arythmies et à une constipation ou une diarrhée.

194. 4 Ni les techniques de respiration profonde, ni les compresses froides cervicales ne constituent une réponse
efficace à ce problème.
situation d'urgence. Bien que la ceinture abdominale puisse être un facteur contributif, elle n'est pas la cause
primaire la plus probable. Le scénario de ce patient est révélateur d'une dysréflexie autonome, un réflexe
pathologique. La cause la plus fréquente est la distension de la vessie ou la rétention urinaire, qui est probablement
due à l'obstruction d'une sonde urinaire.

195. 3 Le tibialis posterior fléchit le pied et l'inverse. Ce modèle nécessite l'action spécifique de ce
muscle. Les autres modèles ne le sont pas.

196. 4 Les options 1, 2 et 3 sont typiques de la lombalgie, et aucune ne nécessite à elle seule une consultation
immédiate. Selle
L'anesthésie autour du périnée est un signe du syndrome de cauda equine et nécessite généralement un traitement
immédiat par une personne autre qu'un kinésithérapeute.

197. 2 La douleur douloureuse est transmise par de petites fibres de type C non myélinisées.

198. 2 Une restriction capsulaire apparaîtrait dans les deux mesures. Sur la photographie A, la flexion du genou
est plus importante.
se présenter avec la hanche fléchie. Dans cette position, le rectus femoris est détendu au niveau de l'articulation de
la hanche, ce qui permet une plus grande amplitude de flexion du genou. Sur la photographie B, le muscle droit

PEAT
4
48 Réponses

fémoral est étiré à la fois sur l'articulation du genou et sur l'articulation de la hanche, de sorte qu'une tension dans le
muscle droit fémoral limiterait la flexion du genou. La photographie B montre également une flexion de
l'articulation de la hanche. La faiblesse des ischio-jambiers n'affecte pas l'amplitude passive des mouvements.
L'étroitesse du mastus medialis aurait une incidence sur les deux mesures.

199. 3 Le monofilament de 5,07 fournit la force la plus faible qui puisse être ressentie par les patients ne
possédant qu'un seul
sensation de protection intacte. La capacité à ne percevoir que le monofilament de 6,65 indique une perte de
sensation protectrice. La capacité à percevoir les monofilaments 3,61 et 4,37 indique une sensation tout à fait
normale.

200. 4 Le système nerveux sympathique est responsable des ajustements en préparation aux situations d'urgence.
(lutte ou fuite). La stimulation du système nerveux sympathique entraîne une augmentation de la fréquence
cardiaque. Le flux sanguin serait détourné de la peau, et non vers elle. Les niveaux de glucose dans le sang
augmenteraient, et non pas diminueraient, en préparation d'un travail accru. Les bronchioles se contractent et ne se
dilatent pas.

PEAT
4

Das könnte Ihnen auch gefallen